1 - Wharton IB Guide

Published on December 2016 | Categories: Documents | Downloads: 60 | Comments: 0 | Views: 288
of 108
Download PDF   Embed   Report

Comments

Content

Wharton Finance Club, 2007-2008

Investment Banking Resource Guide
“DON’T PANIC” -- The Hitchhiker’s Guide to the Galaxy

Wharton Finance Club, 2007-2008

TABLE OF CONTENTS
1 2 3 INTRODUCTION “I HAVE NO IDEA WHAT I’M DOING” INVESTMENT BANKING IN A NUTSHELL
3.1 3.2 3.3 3.4 What is Investment Banking? Major Players Internship Structures Common Associate Tasks and Roles

4 5 7
7 9 11 11

4

RECRUITING COMPONENTS
4.1 4.2 4.3 4.4 4.5 4.6 4.7 4.8 4.9 4.10 Time Table High Touch / Low Touch Employer Information Sessions (EIS) Closed List Events Days On The Job Informational Interviews Cover Letters Resumes Networking (Phone Calls and Emailing) Additional Best Practices

13
13 16 17 19 19 20 20 21 22 23

5

THE INTERVIEW PROCESS 5.1 5.2 5.3 5.4 5.5
5.6

24 24 25 25 26 28
31
Investment Banking Resource Guide Page 2 of 108

The Interview Schedule The Bidding Process Considerations For Interview Scheduling Interview Types What Are Interviewers Really Getting At?
Grade Non-Disclosure

Wharton Finance Club, 2007-2008

6

INTERVIEW PREPARATION 6.1 6.2 6.3 6.4 6.5 6.6 6.7 6.8 6.9 Making Your Story / Pitch Anecdotes Company Research: Knowing Your Audience Knowledge Base Preparations Before DIP Week Tips for DIP Week The Interview Rhythm Calming Your Nerves Body Language

32 33 33 33 35 36 37 39 39 40 41 42 43 46 47 47 47 48 48 50 50 52 52 55 57 67

6.10 International Students 6.11 Practice Interviewing 6.12 Dress 6.13 Final Words of Wisdom 7 USEFUL RESOURCES 7.1 7.2 7.3 7.4 7.5 7.6 8 Text Books Classic Books Wharton Resources Websites – Finance, Business and Industry Websites – Recruiting Related Journals – Business and Finance

INTERVIEW QUESTIONS 8.1 8.2 8.3 8.4 The Basics Other Very Likely Questions Behavioral Questions Technical / Quantitative Questions

Investment Banking Resource Guide Page 3 of 108

Wharton Finance Club, 2007-2008

1 INTRODUCTION
So you’re interested in investment banking? Good for you. We’re here to help. First the good news: Investment banking is one of the most interesting and rewarding careers in finance. Bankers travel the world, interact with the most senior people in business, work with highly talented peers, and constantly take on new projects in new industries. The banker is an analyst, a strategist, a salesman, and a negotiator. Banking is a relentlessly client focused business that is driven by the executions of deals. Bankers want to be exposed to as many deals, the biggest deals, and the best deals possible. They want to work at the top of their game alongside colleagues who excel at what they do. As a career path, banking is a “stem cell.” That is, starting you career at an investment bank opens up many different paths ahead of you within finance or elsewhere. It provides you an opportunity to develop a wide range of skills, take responsibility early, and become part of the Wall Street deal flow. Here’s the bad news: the recruiting process is long and intense. It involves a great deal of self-reflection about who you are and what you want. It also requires successfully navigating a multipart process to ensure that you can then get what you want. This guide is your starting point. Through the years it has been passed from one generation of Wharton students to the next, constantly being updated and improved. There is a lot of information, but the guide has been painstakingly edited to ensure that all the information it contains is current and pertinent. Between now and the moment you have an offer letter in hand, these pages will be your compass, guide book, and safety blanket. This guide, along with the Finance Club’s various educational events and mentoring activities, the MBA Career Management Office, and your Wharton education should leave you well prepared to succeed in your quest for summer and full-time employment.

Investment Banking Resource Guide Page 4 of 108

Wharton Finance Club, 2007-2008

2 “I HAVE NO IDEA WHAT I’M DOING”
As stated previously, DON’T PANIC. You are a Wharton student. Inevitably this means that you are intelligent, hardworking, and obsessively well prepared. You probably hold yourself to higher standards than other people do – and that’s the reason you’re a success. That being said, Wharton will quickly inundate you with an untenable combination of heavy academic lifting, social events, and a busy recruiting calendar. You need to find your way. Here are a few thoughts as you begin your journey: Use your time wisely. You will quickly find that you are limited to 24 hours a day -- despite the fact that you have 30 or 40 hours of activities you’d like to fit into each day. You have to pick and choose to find a pace that works for you. You must also schedule strategically to ensure that you don’t get behind. Use the Finance Club’s resources. We provide lots of resources on our website and a number of educational events for your benefit including Training the Street’s valuation workshop (which should be considered mandatory for those interested in banking). The club will also provide opportunities for you to interact with and get advice from Second Years. All of your coursework is important, but Accounting and Finance are the most important for banking. Use your judgment in prioritizing your school work, but keep in mind that failing to learn your cash flow statement or the leveraging and unlevering of Beta will come back to bite you in the interview. Spend some of your time following the markets on a regular basis. Ideally you’d read the Wall Street Journal every day. But you’re busy. At a minimum, read “Breaking Views” on the back of the Money and Investing Section and check out the “DealBook” section in the New York Times. The technical aspects of banking are accounting and finance, but client interaction is often heavily focused on what’s going on in the market. Your coursework will increase your fluency in the academic areas, but you need to develop your speaking skills in the language of Wall Street. Work with your classmates. Planning to tell your interviewers how great you are on teams? Well start early. Work with people you like, who have similar goals, and who are willing to share information. You can divide up tasks in doing bank research, preparing technical questions, and doing mock interviews with each other. Most successful teams are in the 3-4 person range.

Investment Banking Resource Guide Page 5 of 108

Wharton Finance Club, 2007-2008

As you go through the recruiting process, concentrate on four key themes: 1. Know your story 2. Form relationships 3. Acquire firm and industry knowledge 4. Master likely interview questions

Getting a job in banking is more like asking someone out on a date than taking a final exam… The human element trumps all else.

It is no accident that knowing your story and forming relationships are prioritized above technical questions and even industry knowledge. At the end of the day, banking is a deeply personal business. Getting a job in banking is more like asking someone out on a date than taking a final exam. You can’t win just by studying the practice exams. The human element trumps all else. All the industry knowledge and technical mastery in the world will be inadequate if you can’t clear the hurdle on the interpersonal side. But first: What exactly is

Much more about all of this in the pages that follow. investment banking?

Investment Banking Resource Guide Page 6 of 108

Wharton Finance Club, 2007-2008

3 INVESTMENT BANKING IN A NUTSHELL
This section provides a brief introduction to the investment bank. For more information, please refer to the resources listed in the “Useful Resources” section.

3.1 What is Investment Banking?
An “investment bank” is a catchall term that refers to organizations that do some combination of the following functions: Provide strategic, financial and valuation advisory services Raise capital through the issuance of equity, debt, or hybrid securities Advise companies in merger & acquisition and restructuring deals Offer specialized products and services that satisfy the needs of corporate and government clients Essentially, the bank is an intermediary between those who have money (investors) and those who try to put it to use (clients/corporations). Some banks also have private equity investment arms (making banks the investors) and private client services businesses (helping wealthy individuals invest).

Investment Banking Resource Guide Page 7 of 108

Wharton Finance Club, 2007-2008

Most Wharton students will work in one of the following three general areas: Investment Banking, Capital Markets (fixed income/debt and equities), and Research.

Traditional Investment Banking

Sales & Trading

Research

• •

Capital raising Debt Equity Strategic Advisory M&A Restructuring Takeover defense

• • •

Distribution and execution arm Sells and trades stocks and bonds Manages the firm’s risk and makes markets for securities the bank underwrites

• •

Analysis and recommendations of stocks and bonds Includes company coverage and sector coverage

The three roles, although different, are heavily interrelated. In a simplified example you have a company that wants to go public. Research analysts provide coverage of industries and companies and help investors process information relating to its businesses. Bankers work on the valuation of the company and facilitate its registration and execution. Capital markets actually prices the security and supports its trading. These same relationships work on a variety of other levels. The primary work product of investment banks usually takes the form of preparing “discussion materials” or “pitchbooks” which address the needs of clients and possible solutions for those needs. This ongoing conversation with clients comes at great expense to the banks with no other reward than the hope that the relationships built will eventually lead to fee-generating business. Once a client has engaged a bank, the project moves from pitch to deal as the bank works to execute the deal (whether it be a merger, acquisition, equity offering, debt offering, restructuring, or something else). Large investment banks typically organize their corporate finance functions along product lines (such as Mergers & Acquisitions, Leveraged Finance, and Equity Capital Markets) and industry groups (such as Healthcare, Retail, Industrials, Real Estate, Transportation, Media and Telecommunications, and Technology, etc). Smaller-scale and boutique banks may focus in on particular products or industries.

Banking Dichotomies: Corp Fin vs. Sales Bulge vs. Boutique Product vs. Coverage

Investment Banking Resource Guide Page 8 of 108

Wharton Finance Club, 2007-2008

Things to consider when recruiting for investment banking: Are you interested in capital markets? Do you enjoy working on and managing processes and multitasking? Are you comfortable with accounting and financial concepts? Do you enjoy working with these concepts daily? How much training do you require? In the beginning of your career? Ongoing? Initially, are you willing to give up control over your schedule and personal life? Do you subscribe to the investment banking lifestyle (client service)?

3.2 Major Players
The major players in investment banking can be classified into three general types, depending on breadth of services and size: Bulge Bracket, Large, and Boutique. • Bulge bracket banks recruiting on campus include Bank of America*, Bear Stearns, Citigroup*, Credit Suisse, Deutsche Bank*, Goldman Sachs, JP Morgan*, Lehman Brothers, Merrill Lynch, Morgan Stanley, and UBS*. Large banks recruiting on campus include CIBC World Markets, HSBC*, Lazard, RBC*, Rothschild, and Wachovia*. Boutique banks recruiting on campus include Barclays, Evercore, Greenhill, Houlihan Lokey, Legg Mason, Miller Buckfire, and Sagent. * Bank is also major retail bank as opposed to pure investment bank Banks measure their performance in a variety of ways including revenues, profits, and market share. But the most widely talked about metrics are the league tables, which can be found on Thomson Financial. Banks torture the data in these tables to make themselves #1. For example, “Bank X leads the league tables in announced North American debt offerings between one and two billion dollars.” The next page offers the most up-to-date league tables of completed, worldwide transactions this year. Each bank has its own strengths, be it in executing certain products or in having industry groups with particularly strong client relationships. Many banks, particularly the non-bulge bracket banks, target specific clientele or specialize in certain product and industry groups only. They may not appear at the top of the league tables, but they are worth considering as you evaluate firms.
Investment Banking Resource Guide Page 9 of 108

• •

Wharton Finance Club, 2007-2008

Global M&A League Table

Global Equity League Table

Global Debt League Table

Investment Banking Resource Guide Page 10 of 108

Wharton Finance Club, 2007-2008

3.3 Internship Structures
Just as different banks have different organizational structures, they also have different structures for their summer internship programs. Some banks place students directly into industry or product groups, others have a generalist pool in which summer associates sit together and work on projects from various departments, and a few have rotational programs in which summers are placed in two or more groups over the course of the summer. Each program also has a different structure for the full-time program. Again, direct placement, generalist, or rotational. These structures are not necessarily the same as the structure of the summer program. There are advantages and disadvantages to each arrangement that are great topics of conversation with recruiters at EIS’s. They are also worth keeping track of as you make your decisions about which banks to target and to demonstrate to recruiters that you have done your homework on their banks. By the time closed-list dinners come around you don’t want to be the person asking the head of the recruiting team (who you’ve met three times) whether the program is direct placement or rotational.

3.4 Common Associate Tasks and Roles
The Associate is the second most junior member on a banking team, after the analyst. (Of course, during your summer the full-time analysts will actually be your superiors in terms of knowledge and skills). Nonetheless, a relatively low position in the hierarchy does not diminish the role: the Associate is the project manager and analytical overseer for the “Managing the process” is daily work of investment banking teams. The Associate drives the information flow both internally and externally and has ultimate accountability for the work product being used by senior people and corporate decision makers. The catchall description for these roles is “managing the process.” The entire point of the recruiting process for banks is to find people who will excel in the role of the Associate and who demonstrate the potential to eventually grow into a more senior person at the firm who can take the lead in creating external relationships and generating business.

the shorthand for the role of the Associate. The Associate is the hub.

It is often said that there is no “average day” in the day of the Associate. There are, however, some broadly defined tasks and roles which are summarized on the following page. If you want to see some of the various “directions” in which associates are pulled, the Vault Guide offers a sample schedule.
Investment Banking Resource Guide Page 11 of 108

Wharton Finance Club, 2007-2008

Tasks • Understand financials (I/S & B/S), operations, competitive landscape, macroeconomic conditions Know valuation methods Accretion/dilution analysis, debt sizing models Creating of pitch books Drafting information memorandums Preparing presentations for clients • • • •

Roles Determine what analysis is required Check models worked on by analysts and do some of the more complicated modeling Uncommon to create from scratch, but sometimes perform your own analysis

Valuation and Analysis

• •



• • •

Look through sources for company information (10K & 10Q, credit ratings, research reports, press releases, etc) Communicate with senior bankers for feedback (often very iterative and involves a lot of e-mailing) Put together analyses and presentations based on direction from senior bankers Coordinate with other groups to gather data and synthesize into coherent analysis Coordinate with production resources to deliver endproduct on time Quality control Coordinate meetings Take detailed notes during meetings Answer questions

Communication and Materials







• • Attending internal meetings Attending client meetings • • •

Meetings



Investment Banking Resource Guide Page 12 of 108

Wharton Finance Club, 2007-2008

4 RECRUITING COMPONENTS
Banking recruiting is a marathon. If it’s not painful, chances are that you are doing something wrong. The upshot is that most of those who are diligent, attend the events, and network effectively will get jobs. But, as in a marathon, you must take recruiting one step at a time -- and pace yourself. Work on your resume and your story first. Then, over the course of October and November attend the firm events that are open to all students. Develop the relationships in that time that are necessary to get invited to the closed events later on. Your preliminary work is to make meaningful connections at the firms you are interested in. Most firms have teams focused on recruiting at Wharton (usually Wharton alumni). These are the people with whom you need to develop relationships. Each firm will receive dozens (even hundreds) more resumes than they can interview. You need to make yourself known to the bank to ensure that you earn an interview spot. All of your work up to and including the submission of resumes & cover letters in December is to make the interview list. Attending events is important. Many banks track their interaction with candidates in a database, noting the questions the student asks and the overall impression the student made. Once you get the interview, the game resets, and your performance in the interview will determine whether or not you get the job.

4.1 Time Table
Phase 1:
1st Years
September – October November – January January – May

Phase 2:

Phase 3:

- Training Sessions - Closed List Events - Cover Letter/ Resume Drops - Employer Presentations - Resume Book Submission - Days on the Job/Treks - Interview Prep - Form Interview Prep Teams 2nd Years
September – October October – December

- Interview Prep - DIP Week and post-DIP Week recruiting

-

Employer Presentations - Interview Prep Closed List Events - Interviews Cover Letter/Resume Drops Interview Prep

Investment Banking Resource Guide Page 13 of 108

Wharton Finance Club, 2007-2008

September • • Get the resume done. Work on it a lot. Run it by Second Years you trust. Start thinking about your “story” o Do not underestimate the importance of your story – potential employers want to know exactly what you are interested in and the steps you have taken to get there. Your story is about who you are, something of the utmost importance to people who will trust you with their businesses. Even if you don’t have an airtight “story,” make sure you can create one that is supported by your resume. Your story is personal. It can’t be borrowed from someone else. You need to convincingly explain why you want this job, why your background will help you succeed at it, and why you want to work at this firm. There is an expectation on the part of the banks that you are making a commitment. “I want to try it out” is a sure way to get dinged.

o

o



Familiarize yourself and use the resources provided by the Finance Club, including the investment banking recruiting presentations and guides, mentor program, resume reviews and interview workshops.

October • Attend employer presentations o It is critical to make a few key contacts at the firms you are targeting; follow up initial meetings with emails and phone calls. These people will often become your advocates within the firms. It can be hard to get much time with the recruiting team when they are surrounded by dozens of students each trying to get airtime at these cocktail receptions. Just listening to them can be very effective if you get their business cards. Follow-up with a concise email and ask your questions then. Assume they will save your emails in their database. If you are not sure what to ask about, just listen to the questions your classmates ask until you begin to develop your own understanding of the industry.

o

November • Continue to attend employer presentations.
Investment Banking Resource Guide Page 14 of 108

Wharton Finance Club, 2007-2008



Keep in touch with contacts/establish new ones. Alumni are your best bet. Ask the recruiters to put you in contact with alumni in a department that you are interested in. Bankers are busy people. Alumni will usually take the time to get back to you and share their experiences. Use each conversation as a way to refine your story. Begin to research the firms you are interested in (see example fact sheets at the end of this manual); forming a team to divide this task is a great idea. A team of 4 to 6 can be very effective if each member takes responsibility for researching 2 or 3 firms. Attend “Days on the Job” sponsored by the Finance Club; again, meet as many professionals as possible. Follow-up with emails and thank you notes, where appropriate.





December • Submit resumes and cover letters to the firms you are interested in. If you are a career switcher, cast a wide net and submit resumes to as many firms in your chosen industry as possible. You probably won’t get selected to interview for all of them.

January • • Review possible interview questions in the later sections of this guide. Take part in mock interviews with Second Years to optimize your interview performance. Then mock interview each other to prepare for doing multiple interviews back to back on game day. Mock interviewing is the most important preparation you can undertake.

February • Interview period: Be confident & enthusiastic. First rounds are usually during the first 2 days of the interview period. Firms will call you after your interview during those first 2 days with a decision about the next round. Second rounds are usually during the 3rd and 4th days of the interview period. A few firms have third rounds which they hold at their offices. Follow-up interviews with thank you cards to your interviewers – especially important for the firm you are joining, but generally good practice to develop good will. You may be interviewing with these same people next year.



Throughout the process, it is essential to pace yourself!
Investment Banking Resource Guide Page 15 of 108

Wharton Finance Club, 2007-2008

4.2 High Touch / Low Touch
Wharton students have made it a tradition to categorize banks according to their “touch.” Touch refers to how much contact the bank’s recruiting teams seem to expect candidates to make. Citigroup, for example, has held a large number of events in recent recruiting years and expected candidates to use phone calls and trips to New York to develop relationships with a number of people on their recruiting team. Goldman Sachs and Morgan Stanley, on the other hand, have taken a lower touch approach. The recruiting strategies of each firm may change from year to year depending on the recruiting team, but it is helpful to get a sense about which banks you are interested in and what their approach to recruiting seems to be. Decide your recruiting strategy early on. Is it focused on a few banks or are you really going to try to hit all of them? Recognize that each bank will have at least 3-4 events. If you’re targeting 10 banks, that can mean 40 events during the three months of recruiting. Early on you may have a broader strategy and find over time that it tightens: maybe you just didn’t connect with anyone or at some point you actually grew to get a bad feel for a few banks. Drop them until you have a group of top choices and back-up banks that makes you comfortable. You do not need back-up banks for your back-up banks. Most banks will come to campus multiple times (8x on average) although some come only once or twice. It is highly recommended that you try to attend each event for the banks that you are interested in, especially if they are a high touch bank.

High
Bear Stearns Citigroup Lehman Brothers UBS

Medium
Credit Suisse JPMorgan Merrill Lynch

Low
Banc of America Deutsche Bank Goldman Sachs Morgan Stanley

Preferably attend all events Networking outside of events is highly encouraged

Try not to miss any events Some networking outside of events can be beneficial

Attendance at events matters less (other factors may be weighed more heavily)

Investment Banking Resource Guide Page 16 of 108

Wharton Finance Club, 2007-2008

4.3 Employer Information Sessions (EIS)
EIS’s are the initial point of contact banks make with students. They are also a toehold for your own thinking about recruiting and for establishing relationships with the recruiting teams. Generally, representatives from divisions seeking to hire summer associates will be present at a firm’s first EIS. Eventually, many firms will have separate EIS’s for each division in order to have a more targeted effort. You can expect anywhere from 100-200 students to attend the EIS’s of the larger investment banks and roughly 40-80 students to attend the EIS’s of boutique investment banks. You goal at each EIS is to get to know how that particular firm does investment banking and what are the firms’ strengths and weaknesses. The EIS is a great time to find out what you can say when later asked in an interview “Why do you want to work for us?” A good practice is to bring an index card on which to make notes you can refer to later in preparing for interviews. Do not think of an EIS as a race to collect the most business cards or meet the most people. Recruiting is all about winning “memory share” – what is the chance that at the end of the night someone at the firm will know you were there? Focus on the quality of the interaction and whether you are finding out the information that you need. This will pay off bigger dividends than simply meeting a large number of people. During the EIS, you will need to practice your 30 second story on “why investment banking” and “why me”. The event is about the art of conversation: your personal story, your strengths your interests and goals. As you attend more EIS’s, you will learn more about investment banking and you will begin to tailor and sharpen your story. At the same time, you shouldn’t be doing most of the talking at an EIS. This is an opportunity to ask thoughtful and appropriate questions that you want to have answered to help you decide which banks are right for you – and to use later on in the process. As the process continues, your questions should increase in sophistication as you begin to ask not only basic questions but questions about deal announcements and industry trends.

Recruiting is all about winning “memory share”… Focus on the quality of the interaction and whether you are finding out the information that you need.

Investment Banking Resource Guide Page 17 of 108

Wharton Finance Club, 2007-2008

Keep in mind that unless you happen to have a great conversation with someone, spending approximately 20 minutes with each banker is more than enough. Once you feel that you have asked a few questions, and perhaps received a business card, do not feel compelled to monopolize the banker’s time by standing around awkwardly while trying to think of something else to ask. Having meaningful interactions with 2-4 bankers per event is the goal. But remember: meaningful contact is not standing silently in a circle around a recruiter. If a group seems too crowded, more along to the next group. Do not try to talk only to the senior people present. associates and vice presidents as well. • • Spread your time among

Associates have a greater understanding of what the Associate role is like, and this will give you a better idea as to whether this firm is a good “fit” for you. Senior bankers travel 3-4 days a week and sometimes more. This makes it hard for them to respond to email inquiries (even with Blackberries, a senior banker will not respond to a recruiting email if he or she is in the middle of a meeting). If all of your contacts are senior bankers, the chances of getting a response may diminish. Associates and vice presidents tend to be in the office much more and are therefore more likely to be able to answer emails and have phone conversations. The best time to get people is often outside of normal business hours. Associates are often the only ones left in the office who can have a phone conversation with you during those times. Last but not least: associates on the recruiting team usually draw up the first draft of the interview list!





A final note on EIS behavior: some people become notorious for their willingness to elbow other people out of the circle around senior bankers. This is never a good strategy. The EIS is an opportunity to make an impression on a bank – possibly a bad one. Your Social skills and emotional intelligence are the key. Use them. Bankers are generally very observant people. Acting in a socially undesirable way at an EIS runs a strong risk of branding you as undesirable not just to your classmates but to recruiters.

The EIS is an opportunity to make an impression – possibly a bad one. Your social skills and emotional intelligence are the key. Use them.

Investment Banking Resource Guide Page 18 of 108

Wharton Finance Club, 2007-2008

At the end of the day, EIS’s are a lot of work. You may get out of class at 3:00 pm or later. Then you will have learning team meetings, club meetings, and personal items to tend to. Once you’ve finished this, you’ll have to attend EIS sessions (sometimes several back to back), and you won’t reach home until late. At this point you will want to send thank you emails (which invariably take longer than you think they will), and then you will have school work to do. You will probably see your work pile up day by day as you fall more and more behind. In addition, at some point you will likely become frustrated and have a bad day where you do not feel like you connected with any bankers at an EIS. This happens to everyone. Do not let it get you down. Just remember that you will likely do better the next day.

4.4 Closed List Events
Closed list events are events that are invitation only. Your goal in going to EIS’s and following up with phone calls is to get invited to closed list events and, ultimately, on the interview list. In closed list events, banks will invite a smaller group of people to dinner or drinks and give you a chance to get to know them in a smaller setting. As with other recruiting events, these are evaluative. Not making a closed list event does not mean that you not going to get a job with a particular firm. Nor does making the list mean that you will definitely get a job. It is simply another step in the process. Many firms have more than one closed list event and typically try to invite different students to each event. At closed list events, the goal is to have a more meaningful conversation as opposed to just a standard Q&A session. This is face time, so make sure you talk to the bankers and move around from table to table if possible. Getting invited to these events means you have already made some efforts with the bank: refresh your knowledge of the basics and the deals, mention the people you have spoken with before, and greet people by name if you remember them. You should be careful not to monopolize any recruiter’s time. As always, you are the driver of the conversation.

4.5 Days On The Job
A “Day on the Job” (DOJ) is supposed to be an educational event, typically held at the firm’s NY offices, that give you more information on what it is like to work for them. Unfortunately, sometimes DOJ’s turn out to be nothing more than a glorified EIS held at a hotel in New York that could just as easily have been held in Philadelphia. Nonetheless, you should make an effort to attend the DOJ’s for the firms that you are interested in. While there, make the most of the opportunity to interact with recruiters. Many times, two firms will have DOJ’s on the same day, so you are making a trip to NY to visit more than just one firm.

Investment Banking Resource Guide Page 19 of 108

Wharton Finance Club, 2007-2008

4.6 Informational Interviews
Informational interviews (often just referred to as “informationals”) are a way for you to visit a firm and have a one-on-one conversation with bankers of all levels. Be aware, however, that these types of interviews are designed to convey information to you about the firm and or group of your interest, they can turn into difficult real interviews. You should not schedule an informational unless you feel prepared to go into a real interview. Depending on the banker you meet you may enter a casual conversation or be grilled on topics such as finance, accounting, current business events or deals (especially any that the firm worked on), your past work experience, etc. Most informationals involve meeting multiple bankers (anywhere from 4-7) in back to back one-on-one sessions. It can be a good idea to do an informational, but only if you are prepared to discuss anything on your resume, answer technical questions, and field all other topics that would be fair game for a real interview. If you are not confident in your ability to do this, you run the risk of doing yourself more harm than good. In addition, if you schedule an informational interview, be prepared with an agenda and list of questions. Do not expect the recruiter to start talking without direction.

4.7 Cover Letters
Most students spend too much time worrying about cover letters. The reality is that a cover letter will virtually never advance your prospects. It must be taken seriously, however, because it can seriously damage your application if it has errors. The best approach is to design a cover letter that minimizes risk. With cover letters, resumes, and other written materials accuracy is of the essence. Read, reread, and read again.

With cover letters, resumes, and other written materials accuracy is of the essence… Read, reread, and read again.

Cover letters should be fairly simple. You don’t need to reinvent the wheel when writing them. Use your cover letter to highlight why you would make a good fit for banking, but that can be quickly tailored for each specific firm to highlight the people you have met during the recruiting process and what unique attributes of that firm appeal to you. For more detailed information, MBACM has extensive resources on their website on writing cover letters.

Most students structure cover letters with three main parts: Introduction: Tell the reader what you are applying for and perhaps mention some of the people to whom you have spoken to.
Investment Banking Resource Guide Page 20 of 108

Wharton Finance Club, 2007-2008

Body: Highlight the parts of your past work experience and education that you feel are the most relevant to investment banking. Closing: Thank the reader for taking the time to review your resume.

4.8 Resumes
During your first month at school your resume is your top career priority. Your resume encapsulates, in one page, everything about who you are and who you want to be. At first glance, a resume appears to be a backward looking document – everything on it has happened in the past. In reality, however, a good resume is a prospectus: a forward looking document that tells future employers about why they want to hire you. While your resume MUST be honest and accurate you have substantial discretion on what to include and what to exclude. The rubric is simple: include that which highlights the qualities recruiters consider important to the job you are applying for. You should have approximately 4-5 people look at your resume and give you comments. While you do not have to incorporate everyone’s suggestions, this will give you an idea of what an “outsider” thinks of you when he or she reads your resume. Second years are a huge resource for resume review. Even investment banking analysts find that Second Years who did summer internships are able to provide them with helpful advice on how to make their resume more appealing to firms. The Finance Club will organize a resume review where you can sign up to have a second year look at your resume and provide feedback. You should also find additional readers on your own.

The resume rubric is simple: include that which highlights the qualities recruiters consider important for the job you are applying for.

The MBA Career Management Office also provides a resume content review service, but most students find that this is NOT enough. Also, keep in mind that a second year who has had success in the recruiting process may be able to give a different perspective than that of career management. MBACM is quite helpful in providing resume resources on their website including the excellent list of 174 “resume action verbs” (http://mbacareers.wharton.upenn.edu/student/resumeinfo.cfm). You should take the resume review process extremely seriously and not wait until the last minute to have people review your resume. This is especially true since many times, a firm will make sure you are interviewed by someone who has not met you before hand. In this case, their first impression of you will be based of your resume. If your resume does not show you in the best light possible, or is sloppily put together, you may not advance in the interview process not matter how much rapport you developed with the firm over the course of the year.
Investment Banking Resource Guide Page 21 of 108

Wharton Finance Club, 2007-2008

Be aware that anything on your resume is fair game for interview questions. Think carefully about what you include and what you exclude. Do you really want to include that you won that college strip poker competition? Also, be prepared to substantiate any claims you make and to discuss in some detail any project or experience you include on Anything on your resume is your resume. If you worked on a banking deal in fair game… Be prepared to 2003, do you remember what the EBITDA substantiate any claims you multiple was? Do you really speak fluent Portuguese? make and to discuss in A note for international students: Be sure to experience you include. have native speakers proofread your resume and cover letters to ensure that they are grammatically flawless. There is very little tolerance for imperfect English in written materials of this level of importance.

some detail any project or

4.9 Networking (Phone Calls and Emailing)
Always send thank you e-mails if you have had a meaningful interaction with a recruiter at an event or on the phone. E-mails should be short: easily read on a Blackberry without having to scroll. A good rule of thumb is to send the e-mail within two days of meeting the banker. It is not advisable to send an e-mail immediately after an EIS, due to the fact that all bankers have Blackberrys and you do not want them to go off in the middle of the night. A good tactic to use when writing a thank you email is to include a question in it. Don’t be too worried if you don’t hear back. For example, you can thank them for their time, and then mention a part of your conversation that you found interesting and follow up with a question regarding that topic. This eliminates the “problem” of simply sending “mindless” thank you, and it gives the banker a more compelling reason to respond to you, rather than just sending a “thank you for your thank you” email. Exercise the discipline to check your e-mails for spelling and grammatical errors. Errors can leave a very bad impression on a recruiter, particularly if you accidentally spell the name of the firm or banker incorrectly! Phone calls to bankers – when made in moderation – can be a good idea in order to signal your interest in a firm/group and to learn more about the firm/group with a one-on-one, focused conversation. Always start the call by identifying yourself with first and last name “from Wharton” and ask “is this a good time for you?” If you get voicemail, leave a message and wait a few days before calling again if you don’t get a return call. Understand that it may be difficult to reach a banker and you don’t want to appear overly aggressive.
Investment Banking Resource Guide Page 22 of 108

Wharton Finance Club, 2007-2008

Once you have someone on the phone, keep in mind that the burden will fall on you to lead the conversation. Do not assume that the banker will provide the content for the call. You should let the conversation flow naturally, but also have specific questions in mind that you would like to ask. You do not want awkward silences to occur during the phone call.

The burden will fall on you to lead the conversation. Don’t assume the banker will provide the content.

As a rough guideline, these calls should last about 20-30 minutes max, but should be longer than 10 minutes. A good idea is to have an agenda of 5-6 questions prepared so that you can ask one at any time should there be a lull in the conversation. This will allow you to achieve this time goal and have a substantive conversation without the risk of monopolizing the bankers time.

4.10
• • • • •

Additional Best Practices

Arrive to events ON TIME and do not leave events early. If you must leave early, sit near the door and do it DISCREETLY. Dress code: ALWAYS follow the specific dress code. When in doubt, remember that the bankers will probably be in suits. Attend every event you have RSVP’d for. If you realize you can’t go for some reason, give advance notice that you cannot attend. Be professional, conversant, and enthusiastic when talking to recruiters. Ask thoughtful questions. Don’t neglect your extracurricular life at Wharton. Not only should you be participating for your own mental health, but you are likely to be asked about it and want to demonstrate your involvement in the community. Know one person in each of your target banks well. Don’t get too intimidated. Remember that the bankers are people too and, if they’re recent Wharton alums, they were in your very shoes not too long ago. DO NOT LIE. The finance world is too small. Saying one thing to one bank and something else to another will be a disaster for you. Go the Finance Conference and utilize treks. International job seekers especially: EuroTrek and AsiaTreks are very helpful.
Investment Banking Resource Guide Page 23 of 108

• • • •

Wharton Finance Club, 2007-2008

5 THE INTERVIEW PROCESS
So you’ve got an interview. Congratulations! You’ve come a long way from the start of the process. You’re already halfway there. But remember, you’re only halfway there. All of your networking got you the interview – but it is a new beginning once you enter the interview room. In past years, some students have had multiple first round interviews and no second round interviews. This is a telltale sign that the interview is an independent variable that deserves considerable attention. In the following sections we will give you a variety of tips to help you schedule, prepare for, and perform in your interviews.

5.1

The Interview Schedule

This year’s DIP week schedule for investment banking will be as follows: Round 1st round 2nd round Final offer Group 1 Monday, February 4 Thursday, February 7 Friday, February 8 Group 2 Tuesday, February 5 Wednesday, February 6 Friday, February 8

The major exception to the above schedule is interviews for locations outside the US (e.g. Asia and Europe). MBACM has given permission for banks to interview students before DIP week in order to accommodate these offices’ travel schedules. Some smaller banks may also begin interviewing later in the week. You will schedule your first round interview(s) well in advance of DIP week via the MBACM’s CareerPath website. Be sure you know this process before you need to sign up for interviews. The second round of interviews is a little less structured. If you progress to the second round, the banks will give you a telephone call. The banks could call you at any time between the end of business on the day of your first round interview up to the night before the second round interview. When banks contact you to arrange a second round interview they are required to offer you the choice of at least two slots. In order to ensure you obtain your optimal second round interview time slot be sure to have your schedule with you at all times during DIP week and keep your cell phone with you and on at all times (except in the interview room during an interview!). Companies that interview during DIP week are not allowed to make any offers before 9am (EST) on the Friday of DIP week.
Investment Banking Resource Guide Page 24 of 108

Wharton Finance Club, 2007-2008

5.2

The Bidding Process

Bidding was instituted for the first time last year at Wharton but has been used at other schools successfully. The intention of the bidding process is to decrease pressure on students to attend all recruiting events and to help them signal to employers which students have a high level of interest. Like the course auction, the bidding process gives students the opportunity to quantify the strength of their interest in an outcome. Anecdotally we are told that firms also liked the process because they liked being able to supplement their closed lists with students with a strong interest in them who might have been excluded from their interview lists. Students have deployed many different bidding strategies. Unfortunately, there isn’t a lot of historical data to help students formulate their bidding strategies. Some students bid on their top priority banks (even those they felt reasonably comfortable about getting an interview with) in an attempt to guarantee a spot. Others students bid for priority banks in which they had an interest but were less confident about getting an interview. Others still used bidding to get interviews with banks they had not worked very hard with during the interview process. Bidding is done through CareerPath and is managed by MBACM.

5.3

Considerations For Interview Scheduling

Once you have received an interview invitation from a bank, CareerPath announces a time when the system will allow students to choose a time slot for their interviews. This usually results in a bit of a free for all as students scramble onto CareerPath in order to get the “best” spots. Since slots fill up so quickly, you need to go into the scheduling screen with a strategy: know three or four different times during the day when you want to schedule the interview. When are you at your best? Establish what part of the day you are at your best and try to schedule your interviews for these times. Ask yourself, are you a morning person? Or do you often suffer a “post-lunch coma”? Some people like to schedule their first few interviews with employers who are lower down on their preference list. This lets students “get into the swing” of interviewing before they have to interview with their preferred banks. You may also learn some useful lessons from these early interviews. Other people like to schedule their most important employers up front because they perceive that they will be more fresh. Think about what works for you.
Investment Banking Resource Guide Page 25 of 108

Wharton Finance Club, 2007-2008

On a different note: please keep in mind that we are one community at Wharton. Therefore, in the interest of student cooperation, if a student asks you to change an interview slot because of a conflict they cannot avoid, try to be accommodating – you never know when you will be on the other end of the request. Manage schedules with location in mind Another aspect to consider while scheduling interviews is the interview location. During DIP week all interviews will be held in Philadelphia with the majority of first round interviews held on campus (in JMHH and OCRS). When scheduling your first round interview you will be able to see the location via the MBACM website. For second round interviews you will be told of the location when you receive the call back phone call. Second round interviews tend to be held in Philadelphia hotels. Take note of the exact location and the travel time between locations before scheduling interviews too close together. Count on it snowing during DIP week and factor that into your travel time. Leave some breathing room In addition to factoring in transit time and scheduling delays, consider trying to buffer additional time between interviews to regroup, gather your thoughts, and review your notes for your next interview. Super day interviews Some banks (typically regional offices and boutiques) might schedule ‘super days’ after DIP week at their office in addition to the regular interviewing process on campus. This could be for one more final round of interviewing / meeting more people from the bank, or could be to choose specific groups for the summer once an offer has been made.

5.4

Interview Types

An interview usually lasts for around 30 minutes. As such, if you have an interview scheduled for an hour, it is likely to be either two 30 minute interviews or possibly three 20 minutes interviews. There is no strict rule about the type of people who interview you in the particular rounds. You are, on average, more likely to be interviewed by associates and VPs in the first round and by VPs, directors, and/or Managing Directors in the second round. But don’t be surprised to find a Managing Director conducting the interview in the first round interviews and for Associates to be involved in final round interviews.
Investment Banking Resource Guide Page 26 of 108

Wharton Finance Club, 2007-2008

The most common arrangement is not for one-on-one interviews but for there to be two bankers conducting the interview. Although uncommon, it is not unheard of for three bankers to be in the interview (more of a possibility in the second round). It is likely you will be interviewed by some people you have met during the recruiting process, but the banks also usually want a “fresh set of eyes” to consider your candidacy, particularly in the second round. As such, expect to be interviewed by people you have never met before. The majority of people interviewing you tend to be Wharton alumni and therefore understand the concepts of learning teams and the MBA curriculum. You can therefore expect to be asked questions about these features of life at Wharton, for instance “what feedback did your learning team members provide to you?” Be aware that if you are recruiting for a bank that gives you a division-specific offer immediately after DIP week that the people from your division of interest will be highly interested in interviewing you. You should therefore have some clear answers in mind as to why you want to work in that particular division. You should also get to know these people well in advance of DIP week. Interviews usually follow a specific format and below are some ways to think about interview experiences: ‘Walk me through your resume’ Most interviews start or end with a complete discussion of your resume. This format is open-ended and allows you to structure your interview and lead the interviewers into aspects of your resume that you want to In the interview you are in the highlight. Review your resume before the interviews and practice summarizing driver’s seat – you should be your major achievements in a coherent driving your interviewer to a and compelling manner. A bad summary place that helps your case. rambles and tries to explain every point on your resume. Make your summary relevant to what investment bankers are interested in hearing about. Remember: in the interview you are in the driver’s seat – you should be driving your interviewer to a place that helps your case. Technical, Behavioral, Fit Much has been made of the “behavioral interview” versus the “technical interview”. Make no mistake: All interviews are behavioral interviews. Put yourself in the shoes of the recruiters. Their job is to give offers to people who are smart, people they want to work with, and people who are likely to end up
Investment Banking Resource Guide Page 27 of 108

Wharton Finance Club, 2007-2008

working at their bank. Technical questions are the “threshold tests” that seek to answer the first question. Everything else is behavioral and tries to get at the heart of the recruiter’s job: do they want to hire you instead of someone else at Wharton receiving the same education?

Make no mistake: All interviews are behavioral interviews.

The banks are looking for candidates who are able to communicate their interest in the firm, their ability to work well on teams and with clients, and their prospects for further growth.

There is no way to determine exactly what an interview will feel like – it depends on each applicant's background and what the firm is looking for. Some firms are more notoriously technical than others. Lehman, for example, has a dedicated part of the second round just on technical questions. At the same time, students interviewing at the same firm with different interviewers – or even the same interviewer – report very different experiences and mixes of questions. Be prepared for all types of questions. Good-cop, Bad-cop This is a rare interview format but has been seen in the past during banking interviews. In this format, one banker will play bad cop with the intention of making you uncomfortable with his/her line of questioning while the other banker will play good cop trying to engage you in a positive discussion while helping you along the way. An interview may also take on a hostile atmosphere with the interviewers asking you questions in an aggressive tone or perhaps cutting you off before you complete your answer. In most cases it is not necessarily because the interviewers do not like you or your responses, rather they are trying to test your interpersonal skills, how you perform under pressure, and the firmness of your own beliefs. Stay calm and collected (remembering it is not personal), but hold your ground. In summary, one can never predict exactly what format, questions or people one will face during an interview and hence it is important to prepare well for any situations you might face.

5.5

What Are Interviewers Really Getting At?

A lot of mystery – and anxiety – surrounds the interview process. Many people wonder: What are interviewers really getting at? There are essentially three major criteria that banks care about: 1. Are you up to doing the job? (Level of competence) 2. Are you likeable and dependable? (Level of fit) 3. Will you seriously consider an offer? (Level of interest)
Investment Banking Resource Guide Page 28 of 108

Wharton Finance Club, 2007-2008

The firm’s impression of you as it relates to these questions will be based on three interrelated buckets of information provided by you: your experience and unique attributes, your transferable skills for the requirements of an Associate, and your cultural fit at the firm.

Your job in the interview is to persuade the interviewer that you can do the job, that you are likeable and dependable, and that you are serious about their firm.

Experience and Unique Attributes. You have a unique story and a unique background. That is what you are selling. In thinking about what you have to say don’t try to copy someone else’s story – or start off from what you think the bankers want to hear. Start with who you are and then move in the right direction. You have had success in your previous career. You have a proven ability to deliver in some way. You have had achievements that signal your leadership and success. These are things that need to be communicated to recruiters in a compelling way. Transferable Skills for the Requirements of an Associate. Everything you’ve done in your life doesn’t relate to banking, but some things most certainly do. Focus on those, perfect the way you talk about them so that they are forward facing for your job in banking, and highlight them in the interview. • What are transferable skills? Just as your resume should be forward looking, so should the skills that you highlight. If, in your last job, you were an expert at creating Java applets, that is not a skill that is likely to come in handy as an Associate at an investment bank. If you did extensive financial modeling in excel, however, that is a transferable skill. As you consider what to highlight in your set of skills always keep in mind the requirements of the job of the Associate. Hard Skills vs. Soft Skills. As it turns out, soft skills are a lot harder than hard skills – and in many ways more important. It’s easier to teach people financial modeling or accounting rules than it is to teach them to have a positive attitude, work successfully in teams, or manage relationships with clients. When thinking about transferable skills, don’t just focus on your hard skills. Focus on your soft skills. What qualities are they looking for? Looking at recruiting materials from a few dozen banks you can compile a long list of qualities relevant to success in investment banking. We have attempted to distill them into basic clusters that will cover most items mentioned by banks. Give some thought to how you demonstrate these qualities when developing your story.
Investment Banking Resource Guide Page 29 of 108





Wharton Finance Club, 2007-2008

Commonly Listed Qualities Excellence Record of success / high achiever Demonstrable intellectual capacity High standards / ambition / drive to excel Presence and Professionalism Inspiring trust / confidence in others (particularly clients) Articulate Good judgment Commercial sense Attitude Pleasant to be around Sense of humor Commitment to learning new things Motivated and hard working Teamwork Corroborative (communicates and works well with others) Respectful (both up and down) Adaptable Leadership Initiative (ability to make contributions and add value) Ability to get results Dependability Attention to detail (“safe hands”) Ability to work hard, multitask, and handle pressure Integrity Cultural Fit. Firms have (or at least think they have) unique cultures. Understand what that is and see if it makes sense for you. If it does, say it. There’s no shame in repeating things you have heard from some people at the firm back to them if it resonates with you. In sum: Your job in the interview is to persuade your interviewer that you can do the job, that you are likeable and dependable, and that you are serious about their firm. Please note: Saying you are serious about a firm is very different from telling them that they are your top choice or promising them that you would accept an offer. Unless you really mean it, NEVER imply that you will accept an offer from a firm. See the practice interview questions section for how to handle this issue appropriately.
Investment Banking Resource Guide Page 30 of 108

Wharton Finance Club, 2007-2008

5.6

Grade Non-Disclosure

On January 18, 2007 the Wharton student body held its annual vote regarding the disclosure of grades to employers. The student body voted resoundingly (94% in favor) to reaffirm the Wharton Graduate Association’s policy of grade non-disclosure. The WGA policy states that “the Wharton MBA student body believes that academic grades must be separate from the recruiting process.” As such, “students, both First and Second years, are to refrain from discussing GPAs or transcripts with recruiters” and “recruiters are requested to refrain from asking students about GPAs or transcripts.” Ultimately, however, each individual student has the discretion to decide whether to disclose his or her grades or not. Last year’s recruiting season was the first year after a faculty-sponsored initiative to reaffirm an individual’s ownership of their grades while simultaneously and explicitly encouraging them to disclose those grades to recruiters. Despite this initiative, Finance Club members found that recruiters overwhelmingly respected the student vote and did not ask students about their grades. Some recent graduates even suggested they would view students volunteering their grades in a negative light given the community sentiment on the issue. Among those banks who did ask about grades, students reported that recruiters were understanding when informed about the student GND policy. The WGA’s GND website elaborates on the history of the policy and the reasons for its implementation. (http://wga.wharton.upenn.edu/gnd/.)

Investment Banking Resource Guide Page 31 of 108

Wharton Finance Club, 2007-2008

6 INTERVIEW PREPARATION
Preparation is key for interviews. Preparation should be honed at delivering your message about your distinctive strengths in a concise manner. However, make sure that you are not so well prepared that your answers seem robotic. Wharton students have a reputation of being very strong interview candidates but the most common criticism is that their responses to interview questions seem too pre-rehearsed and their personalities fail to come across. This is the balance you must strike: a very high level of preparation with an ability to be genuine and animated in your interview.

The interview is a 30 minute informercial you get to star in and direct.

The people who tend to do well in the interview process are those who stay relaxed and allow their personalities to come through while selling their strengths in a concise manner. Being yourself allows you to make a connection with your interviewers. Remember it is not the bank that is selecting you, it is the people on the other side of the interview table so making that connection is important.

Don’t forget: the interview is a 30 minute infomercial you get to star in and direct. Because so much of the business of banking concerns your ability to connect with your clients and promote your services, your ability to sell yourself in an interview is a key measure of your ability to succeed as a banker.

Your ability to sell yourself in an interview is a key measure of your ability to succeed as a banker.

In your interview, you need to be able to focus on a few key themes you want to get across and articulate why you want the job and are a good fit for it. Your starting point for interview preparation is being able to answer these questions honestly: 1. Do you know yourself? (Competency, Interest, Values) 2. Do you know your resume inside and out? 3. Do you know how your skills and previous experiences apply to banking? 4. Do you know the company you’re interviewing with and why you want to work for them?

Investment Banking Resource Guide Page 32 of 108

Wharton Finance Club, 2007-2008

6.1

Making Your Story / Pitch

Every question in an interview is an opportunity for you to control the message you are sending about yourself to the interviewer. Clarifying in your own mind your three to four major marketing points is key to making your answers poignant and delivering a clear and concise message. In addition, in many interviews you will be asked if there is anything else that you would like to tell the interviewers about yourself. This is your opportunity to clearly present your distinctive strengths in one clear answer. Knowing your distinctive marketing points upfront will help you deliver an answer that leaves a lasting impression.

Every question in an interview is an opportunity for you to control the message you are sending about yourself to the interviewer.

When considering your key marketing points focus on how these points fit with the role of an Associate: • • • What are your transferable skills for investment banking? Are there significant accomplishments from your previous job? What is it in your background that distinguishes you?

6.2

Anecdotes

It is useful to think of three or four anecdotes/stories about your previous experience that support your key marketing themes. When in doubt, go to one of your prepared anecdotes. Having these clear in your mind will help you answer questions in a timely manner and avoid uncomfortable silences while you search your mind for an answer. The general format for structuring anecdotes is called SOAR: Situation, Objective, Action, Result.

6.3

Company Research: Knowing Your Audience

Along with all of the other things banks value (positive attitude, team work, etc), the ability to differentiate among banks is considered an important quality in a candidate. Bankers usually have a sense of how their bank is different – or at least perceives itself to be different – and will expect you have figured that out as well.
Investment Banking Resource Guide Page 33 of 108

Wharton Finance Club, 2007-2008

As much as possible you want to be able to tailor what you say to each bank. After all, why would a bank want to hire someone who hasn’t bothered to learn anything about them? What message does it send about you if you are unprepared at one of the most important moments in your business school career? In addition to obvious geographic and market position characteristics, firms have other differing qualities: • Know how the firms are internally structured o o • All have some type of industry and product groups Every firm has a different approach to group placement

Know how the firms perceive themselves o o o o o What are their strengths, what are recent significant transactions? How do they view their own culture? What are they proud of? What’s their ambition? Why do their recruiters work there instead of at a different bank? What are employee demographics?

Many students find it helpful to make a one page summary (cheat sheet) on each bank to be your quick reference guide. It should include information such as the names of the people you met and interesting comments they made during the recruiting process, the name(s) of the recruiting captain(s) for the bank, the deals they have recently been Why would a bank want to involved in and what type of program they hire someone who hasn’t have (summer and full time). These summary pages should be developed over the bothered to learn anything course of the recruiting process starting with about them? the EIS. Once interview time comes around, bring your one page to read over while in the waiting room before your interview with the firm, this will keep the details fresh in your mind (and also make you feel a little less nervous because you are doing something productive). Some things to consider including on your summary sheet for each bank include: • • • • • • Stock price Market capitalization Chief Executive Officer People you’ve met at the firm Major deals of the past year Key points made at the EIS and other company events
Investment Banking Resource Guide Page 34 of 108

Wharton Finance Club, 2007-2008

Also consider how you might answer – in a tailored, specific way - these questions: • • • • Why this firm? How do you see your fit with this firm? What other firms are you interviewing with? Where are we on your list? What concerns do you have about us?

6.4

Knowledge Base

As you head into the interview, one of your biggest concerns is probably the fear that you will be asked a technical question you are unable to answer – but that you “should” know. What are acceptable boundaries on your knowledge? What expectations do recruiters really have of you? Simply stated, you will very rarely be asked a question that isn’t knowable from one of three sources: Training the Street’s Valuation seminar or Accounting 621.

Accounting Topics Accounting 621 is your most important class for both preparing for banking interviews and for your summer (and full-time career) in banking. A list of all the topics that could come up in your banking interview is essentially identical to the syllabus for 621. While you are generally unlikely to be asked questions directly about t-accounts or some of the other very basic accounting material of the course, understanding these concepts is useful in getting at the heart of the matter: the three financial statements. You should have a very clear understanding going into the interview about what the three statements are, how they differ, which line items are found on which statements, and how information from one flows into the other two. In addition, banking interviews also include more advanced questions on the later material of the course including FIFO/LIFO, leases, and deferred taxes.
Investment Banking Resource Guide Page 35 of 108

Wharton Finance Club, 2007-2008

Finance Topics Most Finance topics that will come up in investment banking interviews will be covered in the Training the Street Corporate Valuation seminar. This includes the Capital Asset Pricing Model (CAPM), how to calculate the Weighted Average Cost of Capital (WACC), and how to perform Discounted Cash Flow Analysis (DCF). These concepts are all covered in greater depth in Finance 601. But Finance 601 should not be considered “mandatory” either substantively as a signal of interest for banks. In fact, many common finance questions in banking interviews are included in Training the Street but are not part of the curriculum in 601. These include merger consequences analysis and valuation methodologies like comparable companies and precedent transactions. Do not feel pressure to take Finance 601. Every year many students (particularly those in Health Care and Lauder) choose to take 601 in the Spring and are successful in banking interviews. That being said, taking 601 may give you greater comfort with some of the Finance topics discussed above. Students opting not to take 601 are advised to be particularly diligent in reviewing Training the Street as well as the Vault Guide to Finance Interviews to ensure that they understand basic concepts like the time value of money and the differences between equity and debt. Other Topics Some interview questions can seem a little bit more from left field. They generally rely on a student’s knowledge of what’s going on the financial markets or in the economy. This knowledge will not come from coursework, but can be obtained by putting in some regular effort to keep up with the financial news. Most students have a subscription to the The Wall Street Journal – the banker’s bible – which provides the most detailed and accurate coverage. At the very least, read the New York Times DealBook and the major headlines on CNN Money or the Times business section.

6.5

Preparations Before DIP Week

The following are some of the ways you can prepare yourself well ahead of DIP week: • Career Path – Career Management has published (available for download in the Career Management web-site) a guide for navigating through the “Career Path” web-site. Be sure to read this document carefully and submit resumes and cover letters to banks of your choosing. Also, as previously mentioned, make sure you understand the system fully so you can schedule your interviews with as few complications as possible. Have a clear timetable for your interviews written up/printed out so the information can be at your fingertips during DIP week. Include all the information about location and time.
Investment Banking Resource Guide Page 36 of 108



Wharton Finance Club, 2007-2008



Carry the contact phone numbers for the MBACM front office and the HR contact person of the bank. It is important to call and inform the recruiter if you are running late for an interview (needless to say, you should avoid being late). Once you make the first round interview list, some banks will allocate you a mentor for the interview process. Consult this person to establish who will be interviewing you and find out the background of your interviewer(s) if you have not met them before. Your mentor is also a useful resource to ask about whom you may end up interviewing with in the second round and where the location is likely to be. Visit all of the locations where your interviews are scheduled to be held to make sure you know how to get there. Have lots of singles ($1 bills) for taxis you may need to catch. This saves you having to wait for change when you may be pressed for time. Read the Wall Street Journal or the New York Times Dealbook (or anything that summarizes current events) with particular vigor in the weeks leading up to DIP week. You don’t need to be an expert about everything but you should be at least aware of major events/deals.



• • •

6.6

Tips for DIP Week

The following are some of the ways you can prepare yourself for the interview and some useful tips for the day of the interview: • Make sure you have a good night’s sleep the night before and probably even the few nights before. The interview period is intense and draining and you do not want to be tired, sluggish or not mentally sharp. Count on it snowing or raining heavily that day and prepare yourself accordingly. Women may need to wear outdoor shoes to arrive at the interview and then change footwear. Bad weather will also extend your travel time. Arrive early and, if you don’t already know, check who your interviewer is going to be. All of the interviewers’ business cards will be posted on a board at the interview location. Use some of this time before the interview to memorize the people’s names. It is best to address people by their names once they have introduced themselves to you. Remember this for when you leave the interview.
Investment Banking Resource Guide Page 37 of 108





Wharton Finance Club, 2007-2008



Take a simple black folder/portfolio into the interview (similar to the ones handed out at the Wharton Finance Conference) with some note paper and pen you can use. Also carry spare copies of your resume and a few spare business cards – just in case. Both at the Wharton interview facility and at hotels there is an area where you can leave a larger bag/briefcase and coat. Don’t bring those to the interview room. Do not take your cell phone into the interview room. Leave it in the waiting room in your separate bag. If you need to take your phone into the interview room, make absolutely certain that it is turned off. Ringing or vibrating cell phones during an interview are a sign of inexperience and will be viewed in the most negative light. Don’t drink too much caffeine during the day. The banks often offer tea, coffee, and soda at the interview locations. Be careful not to over-consume and become a nervous wreck. Carry breath mints and an energy bar(s)/snacks in your bag/briefcase. Take along some music to listen to, if that helps you relax. Students have also found it a good idea to have a handkerchief or tissues/Kleenex in their bags/briefcases (also take one into the interview just in case). Sniffling during an interview is not good. Make sure you eat at the beginning of the day and throughout the day to keep your energy level up. You are likely to be nervous and as such you are not likely to feel hungry, particularly in the morning. However, you do need to keep your energy level high throughout the day and eating properly goes a long way to lasting all the way through the day. Keep yourself hydrated. Asking for water in between interviews is acceptable. Going to the bathroom before heading into an interview is also advisable. At the end of an interview, remember to thank the interviewers for the opportunity to interview with the firm and reiterate your interest in the firm. Send thank you emails as soon as you can on the same day as your interview. Decisions are made quickly during DIP week (usually the night after your interview) and a follow up “thank you” email is a simple way to keep your name at the front of their minds.





• • •



• •

Investment Banking Resource Guide Page 38 of 108

Wharton Finance Club, 2007-2008

6.7

The Interview Rhythm

While every interview is unique, there is a certain rhythm that you should understand: The First Thirty Seconds. This is actually the most important part. 80% of your impression will form from how you carry yourself into the room and the first set of comments that come out of your mouth. Be confident and upbeat. “Walk Me Through Your Resume” / “Tell me about yourself” / “How and why did you end up in this room?” / “Why Investment Banking?” These questions all mean the same thing: give me your 60 second pitch/story. Behavioral Questions. Convince the interviewer of your competitive advantage, that is to say: be as personable and authentic as possible. Tell anecdotes. Demonstrate that you know what you’re getting into and that you want to work at their firm. Technical Questions. Keep your answers short and concise. If you don’t understand a question, ask the interviewer to repeat it or elaborate on it. Don’t lose your composure. Talk about a problem out loud and ask questions if you need to in order to engage them. Even if you don’t really know the right answer, try to move in the right direction. The interviewer will want to see how you reason. In some cases, this process of talking and reasoning is ultimately more important than a numerical answer. Don’t give up on a really hard question, even if it seems very frustrating.

6.8

Calming Your Nerves

Being nervous before an interview is a typical human response. There are, however, ways to control your nerves. The following are a few tips for handling nerves before and during an interview: • Practice, practice, practice. Practice is the best way for you to become comfortable with an interview and cure those nerves. When practicing, try to simulate the exact interview environment and experience as much as possible. Know your story. Breathe to relax. You want to breathe from your stomach muscles and not the chest. This is called diaphragmatic breathing - you slowly inhale (5 seconds) using the diaphragm muscles (the muscle below the rib cage) and then exhale (5 seconds). Done correctly, this has a calming effect and no one can tell that you are doing it.



Investment Banking Resource Guide Page 39 of 108

Wharton Finance Club, 2007-2008

• •

Just remember that life will go on after DIP week- it is not the end of the world! Keeping a little perspective will help lower your anxiety level. Stay positive throughout the interview process. Have some confidence (not arrogance) in your ability during an interview – after all you were admitted to Wharton!

6.9

Body Language

Body language is a crucial part of your interview. Studies have shown that as much as 50% of what you communicate when talking to someone else is non-verbal. That includes your eye contact, posture, and gestures. This is something to try to improve during the course of your interview practice and to be aware of during the interview itself. Don’t skim this section lightly. Think about it carefully. Always ask your mock interviewers for feedback on your body language. Posture – – Sit up straight (don’t slouch) Be relaxed (not rigid or uncomfortable looking) Sit in the chair with a slight tilt forward Have an open body to all listeners (no crossed arms)

Face Eye Contact

– Show some facial expressions (though not fear!) - Make sure to smile – Make and maintain eye contact with your interviewers – If there are multiple interviewers, be sure to address your answers to questions to all the people in the room - Do not look at the floor (you look evasive and shifty) - Do not look at the ceiling (you look uncertain/insecure) – Show enthusiasm with your voice (no monotone) - Keep a moderate speaking pace (don’t rush answers) - Vary your volume and speed (be dynamic) – Keep your hands where your interviewer can see them - If there’s a table in front of you, put your hands on it - Use your hands to make gestures to show animation - If you’ve been told you gesture too much in the past, make some effort not to over-gesture too much - Avoid fidgeting/twirling hair/other signs of nervousness - Keep your legs uncrossed - Avoid shaking or other signs of discomfort
Investment Banking Resource Guide Page 40 of 108

Voice

Hands

Legs

Wharton Finance Club, 2007-2008

6.10

International Students For improving your English, there’s no time like the present to start speaking in English socially more often.

International students (particularly those not fluent in English face some additional challenges when interviewing for US based positions. Nonetheless, many students have successfully been able to frame their international experience into a selling point during an interview. The following are some important considerations for international students: •

Practice is even more important. Try to practice with native English speakers (e.g. people on your learning team and friends from activities). Some students make a contract with themselves to only speak English in January or during the weeks leading up to DIP week, particularly when spending time with their fellow countrymen and women. For improving your English, there’s no time like the present to start speaking English socially more often. To gain a better insight into how the interview is likely to run, practice your interview or pitch with people (Second Years) who have had a similar experience to yours (same culture, similar background) in order to receive their tips on how they have marketed themselves. In contrast, it can also be useful to practice with people who have a very different background than yours or come from a totally different culture so that you make sure you are not assuming the interviewer understands your background or the jargon you are using. For instance, don’t assume that the interviewer knows about your university or the education system in your country. If your university is among the best in your country, say so. Providing too few details about yourself may confuse the interviewer. Having said that, giving too many details will bore him/her. Try to read the interviewer from his/her reaction and body language. Striking this balance makes practice all the more important. Become familiar with some of the finance and investment banking terminology. Ask your classmates (former investment bankers) to help you with the jargon. Be ready to market yourself. Americans are notoriously great self promoters while self promotion is often frowned upon in other cultures. Practice marketing your strengths with some of your American classmates to get a feel for the appropriate tone. Be more direct/succinct. Americans have a reputation for being direct and to the point whereas some international students have been criticized for being too long winded with their answers. Avoid putting the answer to a question at the back of an explanation. Preferably answer the question in its simplest form upfront and then provide an explanation.
Investment Banking Resource Guide Page 41 of 108



• •



Wharton Finance Club, 2007-2008

• •

Provide positive rather than negative answers. For instance, “I love Barcelona” is better than “I don’t like Real Madrid”. If you do not fully understand a question ask for clarification. Seeking clarification on a question is much better than providing an answer that does not match the question. Useful ways to ask for this clarification include: “Could you please clarify your question?”, “Could you please be more specific?” or, “Could you please re-phrase the question?” Don’t be apologetic for your accent. If an interviewer has not understood your answer and you think it is because of your accent, simply re-answer making a greater effort to make yourself clear. Make the fact that you are international a selling point for yourself. Most employers are looking for lots of diversity of thought and experience and, as such, want to hear about your relevant international experience. Make the effort to find the right spin for your story and then practice delivering your message with confidence. Don’t be seen as timid or unsure of yourself – particularly as it relates to being an international student. Practice voice modulation when mock interviewing. It is not just what you say but often how you say it. Therefore, practice making your voice sound excited when you are talking about an important point about yourself. Keep your confidence all the way through DIP week. Almost everyone who interviews during DIP week will be rejected by one employer (i.e. get “dinged”). Don’t let this discourage you. Some international students have suggested that their disappointment about being dinged may be more obvious to employers than native speakers of English. Work hard not to lose your confidence. Maintain a positive attitude all the way through DIP week.









6.11

Practice Interviewing

As previously emphasized, practicing is the key to interview preparation. Form study groups – It is highly recommended that students form a small study group of 3-4 students. This group can provide the much needed support during the recruiting season (e.g. sharing information on closed list events, conducting peer mock interviews, resume reviews, etc). In addition, it is a fantastic forum to conduct mock interviews in ahead of DIP week in a non-threatening environment. To get the maximum benefits try to form a diverse group based on nationalities, ethnicities, prior work experience, gender, etc. in order to draw from the experiences of the group.
Investment Banking Resource Guide Page 42 of 108

Wharton Finance Club, 2007-2008

Mock interviews – Career Management conducts a mock interviewing day(s). Be sure to sign up for this event. The Finance Club also holds mock interviews with Second Years that you should also take part in. If you want to, consider bringing a tap to mock interviews so you can listen to yourself later. The following are a few pointers for interview practice: • Practice full length interviews. Investment banking interviews tend to run for about 30-45 minutes. It can be difficult to sustain the energy, enthusiasm, and focus during such a long period. It is, therefore, best to practice for the full length of time so you become accustomed to the pressures of a real life interview. Try to simulate the environment as much as possible. To become accustomed to the interview format and environment try to make the interview as real life as possible. Suggestions include finding a quiet room with a table and chairs (rather than just a couch and coffee table) and sitting directly across the table from your mock interviewer. Practice for different types of interviews or scenarios (nice interviewer/rude interviewer etc.). Also, simulate interviews for different banks so you can practice the “why bank X” question. Delivery is key. You may know the answers to all the questions but poor body language or tone can damage the impression you are trying to make. To gain an unbiased perspective on your interview technique, practice with someone who does not know you very well. It can be useful to do some of your practice in groups of three. One person is the interviewer, one is being interviewed and the third can focus on the feedback for the session.





• • •

6.12

Dress

Follow these tips – and shine your shoes and iron your shirts before the interview. Women • Either a skirt suit or a pantsuit is completely fine for interviews. Traditionally, a skirt suit is considered more conservative but there is absolutely no requirement to wear one. You can't go wrong wearing a solid navy-blue, black or medium-to-dark-gray business suit, with a solid-white or light blue, no-frills dress blouse with a high neck line. Long sleeve blouses are best.
Investment Banking Resource Guide Page 43 of 108

Wharton Finance Club, 2007-2008



Be careful not to wear too much perfume as the interview room can often be very small and it is a common observation that students’ perfume is overpowering. Limit yourself to “one squirt” of your perfume in the morning before the interview. We put this item so high up on the list because it’s actually the top complaint by interviewers who have to spend their whole day in a very small room and who would prefer not to be bathed in your scent. It is highly recommended that you wear hosiery (skin colored is best). Carry a spare pair in your bag, just in case the pair you are wearing tears. Wear low-heeled, conservative dress shoes (closed at the front and back) that are color coordinated with your outfit. Don't go without a belt if you're wearing a skirt or pants with belt loops. Wear a dress belt that matches your shoes (e.g. black with black). Style your hair tastefully or have it professionally done. If it's long, wear it up or back, so you're not constantly flipping it out of the way. Don't wear a hairstyle that covers your face. Stick to discreet hair accessories if any. Avoid fingernail designs and bright- or odd-colored polish. Clear or no nail polish is best. Take a natural approach to applying makeup and avoid unusual or bright colors. Definitely don't apply sparkles to your hair or face. If you wear earrings, wear small, conservative ones. Wear only one per ear in the traditional earlobe position. It is best to opt for pearl earrings and/or necklace since it coordinates best with most conservative business suits. In general, keep jewelry simple, small and discreet. Not wearing any jewelry is also a good choice. Carry anti-perspirant with you if you are worried about sweating. It looks more professional to carry a briefcase or portfolio into an interview than a purse/handbag.

• • • •

• • • •

• • Men •

A navy-blue, medium-to-dark gray, or black business suit is best (discreet pinstripes in these colors are fine too). Avoid green, maroon and light colored suits. Avoid wearing different colored pants and jackets (such as dark gray pants with a blue blazer), but rather invest in a suit as described above.
Investment Banking Resource Guide Page 44 of 108

Wharton Finance Club, 2007-2008



Be careful not to wear too much cologne as the interview room can often be very small and it is a common observation that students’ cologne is over powering. Limit yourself to “one squirt” of your cologne in the morning before the interview. We put this item so high up on the list because it’s actually the top complaint by interviewers who have to spend their whole day in a very small room and prefer not to be bathed in your scent. Long-sleeve dress shirt. Solid white is always a safe option. If not white, light blue is another good option. You are not trying to make a fashion statement so you should be conservative. A conservative silk tie. Silk makes a nicer knot and dimple than most other fabrics. If you can't afford 100-percent silk, try a silk blend. Good tie designs can be found at Brooks Brothers. Your tie should end in the center of your belt buckle, not above the waist or down to the crotch. Wear calf-length dress socks (not crew or tube socks) that match your suit. Wear low-heeled, conservative dress shoes (cap-toe or wingtip) that are color coordinated with your suit. (For example, don't wear brown shoes with a navyblue or gray suit. Wear black.). Make sure your shoes are well polished. Some say that laced shoes are dressier than loafers, but that's a minor detail. The important thing is to wear conservative, polished, color-coordinated dress shoes, not sneakers, cowboy boots, etc. Don't go without a belt if you're wearing pants with belt loops. Wear a dress belt that matches your shoes (e.g., black with black). Get a professional haircut or trim. If you have longer hair and really can't stand the thought of cutting your long hair for interviews, at least tie it back in a ponytail. In short, well-groomed hair is best. Clean-shaven is the best way to go. If you have a beard, goatee or moustache, shave it off for interviews. If you just can't, at least make sure it's wellgroomed. Carry anti-perspirant with you if you are worried about sweating.





• •

• •





Investment Banking Resource Guide Page 45 of 108

Wharton Finance Club, 2007-2008

6.13
• • • •

Final Words of Wisdom
Be yourself Be proactive Focus Don’t stress out

If You Get The Offer: A final, final word of wisdom for you to begin thinking about now: when the time comes for you to decline offers, handle it with grace and dignity. If firms call you, return their calls. It’s not only the right thing to do, but if you are one of the 25% of students who don’t get offers at the end of the summer, the first people you will turn to are those whose offers you refused in the spring. Be on good terms with them. If You Do Not Get The Offer: Recruiting does NOT end with DIP week. You can expect a third of Wharton First Years not to have secured their summer internships by the end of DIP week. The Finance Club and MBACM will be there to assist those continuing their job searches after DIP week.

Investment Banking Resource Guide Page 46 of 108

Wharton Finance Club, 2007-2008

7 USEFUL RESOURCES
There are more resources out there about banking than you can possibly take advantage of. We have tried to pull together some of the more frequently used books, websites, and journals.

7.1

Text Books

Principles of Corporate Finance, Brealey, Meyers, and Allen – Straight up, this is the finance bible. This book is the reference book for financial theory and a great guide as you work your way though the technical questions. Financial Analysis for Managers, Robert C. Higgins – This book can be particularly useful for those without prior finance backgrounds. In straightforward language Higgins sets out corporate financial and valuation principles in an applied fashion and in a way which a corporate finance manager would use them. It is by no means the finance gospel but it demonstrates some clear applications of financial theory.

7.2

Classic Books

Liar’s Poker, Michael Lewis – This book is actually about trading, but it is the single most commonly referenced book on Wall Street. It’s a quick, fun read. At the very least, don’t stop reading until you know what “Equities in Dallas” means. Barbarian’s at the Gate, Bryan Burrough and John Helyar – The Liar’s Poker for M&A types, this tells the story of the greatest LBO battle in history (RJR Nabisco). It’s a great read and particularly relevant given the increasing importance of private equity in the past few years. Monkey Business, John Role – This is the tale of two MBA students, one from Wharton, one from Harvard, who become friends while working as summer (and later fully time) associates at the legendary DLJ. It is not a flattering book about banking, but is often referenced and worth reading. Don’t bring it up during an EIS! Den of Thieves, James Stewart – The authoritative account of the insider trading scandals that rocked the industry in the 1980s. It’s a great read both as a portrait of the times (the swingin’ 80s) and as an introduction to the ways of the Street. The House of Morgan, Ron Chernow – This book will not land you a job but it is a good perspective into the history of banking in America. It is business history but it is good business history. Perhaps read it after you land the job (Morgan Stanley and J.P. Morgan summer associates receive a copy when they start their internships).
Investment Banking Resource Guide Page 47 of 108

Wharton Finance Club, 2007-2008

7.3

Wharton Resources

Spike Calendar – Check it every single day to make sure you’re not missing recruiting events, showing up when they’ve been cancelled, or going to the wrong room when they’ve been moved. MBACM – Make sure you’re meeting deadlines for the resume book and for CareerPath as well as utilizing the many online tools they provide. Finance Club webCafe – Updated constantly with job announcements, descriptions of upcoming events and event sign-ups. Also has an excellent database of investment banking interview questions. There are usually 1-2 presentations on investment banking recruiting strategies that contain valuable tips and additional resources. Wharton’s Lippincott Library – A variety of online resources are available for free via the University of Pennsylvania library. Some of the best “e-resources” found here (http://www.library.upenn.edu/lippincott/) include the following business databases: Factset - provides financial and economic information, including fundamental financial data on companies) Thomson Analytics (formerly First Call) - provides analyst estimates on stocks) Thomson Banker - provides data on a variety of transactions Investext- provides access to research reports from investment banks and brokerage houses) Capital IQ – A favorite for doing company and deal research to follow the market or prepare for interviews. http://mycareer.wharton.upenn.edu/student/capitaliq.cfm#. Juice Job cards – Not really a Wharton resource, but some people have found these useful in doing interview prep. (Others disagree). It might be a good idea to buy this from a second year student or online at www.amazon.com.

7.4

Websites – Finance, Business and Industry

www.wsj.com - Paid print subscribers get free online access to the Journal’s site. It is still regarded as the domestic source of financial information. The site also has decent search features that allow you to research firm coverage and deal activity. The international compliment to the WSJ is the Asia Wall Street Journal.

Investment Banking Resource Guide Page 48 of 108

Wharton Finance Club, 2007-2008

http://dealbook.blogs.nytimes.com – Free site run by the New York Times. Updated daily with useful information on M&A, investment banking, IPOs, venture capital and private equity news. You can sign up for daily emails to be sent to you. www.thedeal.com – This subscription site offers insight into M&A, Private Equity and Restructuring activity. It is a decent source of current deal information. The “Dealmakers” section has particularly in-depth insight into personalities in the industry and gets beyond the firm-specific level. This is helpful in researching keyplayers at each firm. www.thomson.com/solutions/financial - Thomson offers free league tables and is generally considered the definitive source in deal listings. The site also offers complimentary one-time subscriptions to industry journals such as Mergers & Acquisitions and Buyouts. Sign up on the website. www.investopedia.com – Great place to look up finance terms you don’t know. www.mergerstat.com - Mergerstat is another often used source for current M&A activity. Similar to Thomson, many of MergerStat’s research are by subscription only. Note that the site does offer free league tables and rankings specific to M&A activity. www.wallstreetreporter.com - It offers good insights into Wall Street news. Features include IPO, Venture Capital, Private Equity and Legal related content in addition to an array of interviews which typically focus on CEO’s and Wall Street Analysts. www.iimagazine.com - Institutional Investor online site offers the II ranking as the primary attraction. II rankings are regarded on the Street as the measure for a firm’s research analysts. This site also offers a complimentary monthly email newsletter service that contains recent industry developments and excerpts from the magazine. www.euromoney.com - Perhaps the most valuable issue of Euromoney comes in the Euromoney Awards for Excellence issues. Here, Euromoney posts its winners in an array of financial services categories. The website and the print magazine have a distinctly non-U.S. focus. www.feer.com - This is the site for the Far East Economic Review. This Dow Jones publication focuses exclusively on Asia with emphasis on economics more so than finance. There are links for investor articles and capital market information. finance.yahoo.com - One helpful resource that Yahoo! and the array of other finance sites offer is the ability to create a stock portfolio. It is useful to create a portfolio of public firms that you are targeting to monitor their stock performance and earning announcements on a daily basis.

Investment Banking Resource Guide Page 49 of 108

Wharton Finance Club, 2007-2008

finance.google.com – Quickly catching up with Yahoo! as the best free corporate finance site on the web. Has the best share price history charts on the internet and a deal with Reuters for in depth company descriptions.

7.5

Websites – Recruiting Related

The financial recruiting websites offer cursory information about financial services firms and careers in finance. These sources are in no way stand-alone nor comprehensive in nature. Generally, the most current and meaningful information will come from the company itself (e.g. presentations, days on the job) and finance related resources (e.g. WSJ and TSFD). We recommend using recruiting guides to gain some general background about potential careers and the firms. During the interview though, it will be information gathered from your contacts, your firm visits, info sessions and firm specific research that will set you apart. www.thevault.com - The Vault offers a recruiting and job search website that collects information and accounts about firms and job opportunities. The general firm information is decent, however, specific accounts are suspect so use with caution. The Vault offers “Career Guide to Finance Interviews” and the “Career Guide to Investment Banking”. The Wharton Career Management website also offers these for free download. www.wetfeet.com - Wetfeet offers recruiting and job search information similar to The Vault. Wetfeet’s online platform also offers sales of its popular industry (Careers in Finance) and firm guides (firm specific). The Wharton Career Management website offers these for free download.

7.6

Journals – Business and Finance

If you are focused on a particular industry we recommend pursuing journals in that industry. Many of the Wharton Clubs are a great resource in this area (e.g. Tech Club, Travel and Leisure Club, Energy Club). Specific products have specific journals as well (e.g. IPOs and Red Herring, Mergers & Acquisitions, LBOs and Buyouts). The following is a brief list of general finance and business publications that we have found helpful and commonly used in practice. Many of the journals are covered under the website section: The Wall Street Journal, Financial Times, Institutional Investor, Euromoney, Mergers & Acquisitions (TF). Here are some others that are not: BusinessWeek – A widely read source for weekly business and financial information. The editorials and economic analysis are good.

Investment Banking Resource Guide Page 50 of 108

Wharton Finance Club, 2007-2008

The Economist – A well regarded magazine, however it is not capital markets focused. It does provide exceptional insights into global industries and periodic reviews of financial services and the global capital markets. Fortune – This magazine has good managerial insights into industries and the market. Use the magazine’s website (www.fortune.com) to conduct company specific information. The magazine also publishes a list of top Wall Street Research Analysts. Forbes – Similar to Fortune in its managerial insights and lack of capital markets focus. People tend to use this interchangeably with Fortune as a good macro-level magazine with broad industry and geographic coverage.

Investment Banking Resource Guide Page 51 of 108

Wharton Finance Club, 2007-2008

8 INTERVIEW QUESTIONS
Over the course of recruiting you will find a large number of resources for possible interview questions from MBACM, Second Years, the Vault Guide, and the Finance Club online database. Here we have attempted to compile a strong working list from these sources for you to use as your foundation in interview preparation. For the most common and important questions we have provided some commentary or food for thought. Where we thought it would be educational, we have provided answers for some of the technical questions. Where we thought it would be better for you to find the answers yourself, we have provided only the questions. We have erred on the side of the inclusive: that’s to say, we’ve included every question we could get our hands on. The questions are not ranked in terms of difficulty, they are ranked by likelihood of being asked. The more oddball questions we’ve relegated to the bottom of each section and they can be safely ignored. If you spend some time thinking about how you would answer the questions on these lists, you should have very few surprises. As you think about how to spend your time, think about the impact on the margin. Are you better off spending your time improving your pitch and doing more research on a bank or researching the most obscure technical question? (If you’ve read this guide carefully, you know the answer.)

8.1

The Basics

While every interview is unique, there are a surprising number of questions that seem to come up in every interview. These are The Basics and you should be very well prepared for them. 1. Tell me about yourself / Walk me through your resume • Even though they have your resume in front of them, this is how most interviews start. They have one page of information on you, but they want to see what you choose to highlight, how you talk about it, and your presentation in doing so. This is an invitation for you to market your strengths that are relevant to your ability to succeed on the job. Be confident, upbeat, and distinctive. This is also a chance to create a sneak preview of stories you want to tell them – and by highlighting items on your resume you make it more likely they will ask about them. Focus on accomplishments and transferable skills.
Investment Banking Resource Guide Page 52 of 108



Wharton Finance Club, 2007-2008



Restrict your answer to the aspects of your experience that highlight your key marketing points or those that are required to make your answer flow properly. Your answer should not exceed 2 minutes. You don’t have to cover your entire life history. Where you choose to focus tells the interviewer about you. You should be looking at what you learned at various steps in your career and education and why you made the jump to the next step. No doubt you will meticulously script the answer to this question and practice it in many mock interviews. By DIP week you will be bored stiff by your own answer to this question. Be careful of rehearsing this too much that it sounds unnatural.



2. Why banking? • Well, why banking? Be honest with yourself. That is how you will find the best answer to this question. Once you think you have it, go over your answer with mock interviewers and other peers. Anticipate rebuttals to your answers ahead of time and prepare responses to them. Different people will have different reasons for being interested in banking. Some will emphasize the focus on the client, others the analytical rigor and intellectual challenges, others still the people and project management skills. Ask yourself honestly what your reasons are and build your story around those reasons – and your unique experience and personality. This question may also be asked as “Why not consulting/Sales and Trading/Private Equity?” You should be prepared to answer explicitly in the negative why those don’t fit for you based on your experience and your interests and then circle back to what uniquely about banking makes it the right choice for you. You will sometimes be asked what other careers you are considering. Be honest, but if you are only interested in banking be very explicit about that. Money is not a good answer to this question.





• •

3. Why this bank? • • Ask yourself - what are the features of the bank that appeal to you the most? How do the features of the bank match with aspects of your previously jobs that you enjoyed?
Investment Banking Resource Guide Page 53 of 108

Wharton Finance Club, 2007-2008

• •

Did someone say something at an EIS that struck a particular accord with you? Try to answer this question in terms of the ways you best fit with the bank rather than just in terms of what the bank can do for you.

4. Tell me about a time you, as part of a team, succeeded / failed. Tell me about a time you, as an individual, succeeded / failed. • These are four of the most frequently asked “tell me about a time” questions. They provide an opportunity to say something about who you are and how you handle things. This question is really asking about what you know about the role of the Associate and what working with you will be like. They want to know you will work well with senior bankers, peers in different groups in the banks, and the analysts who are technically below you. Put the emphasis on transferable skills / qualities desired in an Associate. For each of these four questions, put together two to three anecdotes. This will leave you well prepared for these questions and other questions that are bolstered by anecdotal responses. Note the distinction between “individual” experiences and “team” experiences. Your anecdotes should be appropriate to the question.







5. We’re concerned about X. Can you explain more to us? • There will be concerns about everyone. If you did banking before it’s “why did you leave?” and “why do you want to come back?” If you worked in private equity or at a hedge fund it’s “are you sure you want to be on the sell side?” and “do you understand the differences between the buy side and sell side?” If you’re a career changer there are a variety of possible “challenges” to your thinking. Anticipate the concerns about you. (They will likely be raised in mock interviews with Second Years.) Then figure out appropriate responses. Sometimes this dialogue can be quite forceful with multiple rebuttals to your arguments and challenges to your sincerity about or commitment to banking. This is part of the process. If you have a well-thought out story and, through mock interviews, are able to anticipate and prepare responses to likely objections, you should do fine.
Investment Banking Resource Guide Page 54 of 108

• •

Wharton Finance Club, 2007-2008



This is often the part of the interview where one interviewer plays the “bad cop.” State your answers with clarity and conviction. Be polite, but firm in holding your ground.

8.2

Other Very Likely Questions

There are a few other very common questions that happen towards the end of the interview that you should also be aware of – and that require extra preparation. 1. Do you have any questions for me? • This is the most common way that interviews end. In some cases, it’s the only question an interviewer will ask. Is serves as the big reminder that you’re in the driver’s seat. Have a few questions prepared. At the end of the interview, there is likely to be an opportunity to ask questions. It is not completely necessary to ask questions but if you have a few questions prepared it might help prevent an awkward silence in the interview. Questions could include asking about the interviewer’s background (so make sure you are paying attention at the beginning of the interview to ensure you don’t ask a question that has already been answered) or asking about the second round process. Here are a few commonly suggested questions for you to ask: Personal experience: How did you get your start in banking? How did you end up in your group? Advice: What makes a good Associate? What are the key factors of success for individuals starting in banking? Commentary: For a more senior banker consider asking questions about market trends and recent deals – but be prepared to have a dialogue. 2. Is there anything else you want to tell me? • This seemingly scary question is actually the best gift an interviewer can give you: the pure play selling opportunity. Seal the deal with this one: why banking, why that firm, why you. If you’re good, you can turn “do you have any questions for me” into “is there anything else you want to tell me.” For example, “Most of my questions have been answered at this point, but I would like to say…”
Investment Banking Resource Guide Page 55 of 108





Wharton Finance Club, 2007-2008

3. If we make you an offer right now, will you accept it? • You probably won’t get asked this in a first round interview, but many people do get asked it in a second round interview. DO NOT SAY YES UNLESS YOU MEAN IT. You do not need to say “yes” in order to get an offer and you are bound by your word once you say it. If you are ready to commit to one firm over all the others, then you may tell them. Otherwise, be prepared to discuss in a thoughtful way how you would evaluate multiple offers including how you would compare spec firms. Think about it this way: Until a firm has made you an offer, they have not committed themselves. As such, short of an offer, you cannot be expected to commit yourself to them. Rather, you just need to discuss the reasons that you are interested in the firm. Showing bad faith in answering this question is very ill-advised. If you tell multiple firms they are you top choice, it will often get back to them. (Recruiters from different firms talk to each other about recruiting.) This can lead to you not receiving an offer from either firm. Even if you get away with it during the interview period, if you subsequently refuse an offer from a firm you have told you would join you create very bad blood. The Career Management office is notified and often other firms are made aware of your actions. This is can have serious adverse consequences for full-time recruiting.





4. Tell me about a deal you’ve been following. • It is not uncommon to be asked in an interview – particularly in the second round – to talk about a deal or two you have been following. You should have several deals you are prepared to talk about comfortably should the question arise in an interview. Students who are expressing a strong preference in an industry group usually follow deals in that area, but even those without a specific group preference should go through the exercise of following a recently completed (or recently announced) transaction. The best source for deals to follow is the Wall Street Journal, although you don’t need to choose a deal that is front page news. You should be able to summarize the deal – and why you think it’s interesting – in a few questions. In order to prepare your summary (and to anticipate follow up questions) consider the following factors: - When was the deal announced? - What is the transaction value? - What are the terms of the deal (cash or stock)?
Investment Banking Resource Guide Page 56 of 108



Wharton Finance Club, 2007-2008

-

Have any revenue or cost synergies been announced? If so, what are they qualitatively and quantitatively? What is the transaction value as a multiple of EBITDA? Of revenues? Are theses multiples in line with recent deals in the industry? Is the deal accretive or dilutive for the acquirer? What is the strategic rationale of the target? For the acquirer? Which banks are involved in the transaction? What has been the market’s response (in terms of stock prices)? What impact (if any) is this deal likely to have in its industry? Is it likely the deal will go through? If not, why not?

8.3

Behavioral Questions

Answers to these questions are meant to understand more about who you are – what motivates you and how you respond to new situations. There are also a variety of questions designed to check on the “airplane test” – is this person interesting enough to sit next to for a long plane ride? Self-confidence and likeability are particularly important in these types of questions. Some of these also call for anecdotes, which you should have prepared. The more anecdotes you have prepared going into an interview, the better. At the very least, you need to sketch out and practice your strengths and weaknesses complete with anecdotes and take aways. A final note: some behavioral questions (“what’s your favorite book?”) may have an easy answer (“Huckleberry Finn”). Don’t wait for the banker to ask you the real question: “Why?” Make your original answer both the short answer (in this case the title of the book) as well as a few sentences of explanation about why something is your favorite. It’s the reasoning that is important.

GENERAL BEHAVIORAL
1. What are your strengths? • This question is really asking why you’d be a good banker. All of the qualities described earlier in this guide are good answers – find the ones that you can talk about best.

2. What are you weaknesses? • Obviously this is a much harder question. You need to have three or four weaknesses that you can talk about. It’s important to be able to demonstrate insight and self-awareness. Admit imperfection, accept responsibility for it, demonstrate that you understand its impact on others, and, most importantly, talk about what you are doing to improve.
Investment Banking Resource Guide Page 57 of 108

Wharton Finance Club, 2007-2008



You should come up with a real weakness, not a strength disguised as a weakness. (People hate to hear “I’m too much of a perfectionist.”) That being said: Never make your weakness a fatal flaw.

3. What achievement are you most proud of? Least proud of? 4. If I were to call your boss, who knows you well, what would your boss say about you? 5. Describe an ethical dilemma you’ve dealt with. 6. You say on your resume that you’re interested in X. Why? Tell me about it. 7. What are the three most important events of your life? 8. What 3 words best describe you? 9. What’s your favorite quote? 10. Who’s your idol/mentor? Other than a relative? 11. What’s your most embarrassing moment? 12. Tell me a joke. 13. If you could be any animal, which one would it be? 14. What three things would you want stranded on a desert island? 15. On what grounds would you dismiss someone? 16. What was the last book you read for fun? What are you reading right now? 17. What’s your favorite book? 18. Most people come in wanting us to know three things – give us seven. 19. Of my colleagues who have interviewed you to this point, whom would you hire and why? 20. Were you surprised that we called you back for 2nd/3rd round interviews? 21. What’s the most controversial idea that you hold deeply? 22. How do you deal with rejection?
Investment Banking Resource Guide Page 58 of 108

Wharton Finance Club, 2007-2008

23. How was your vacation? 24. If you could invite 4 people over for dinner, who would they be and why? 25. Whose personality do you think had the biggest contribution to your own personality to date, your mother or your father? Explain. 26. How do you think your performance in your last job should be assessed? 27. Give me an example of a situation where you displayed good judgment 28. Suppose everyone has a chip on his shoulder, big or small. What’s yours? 29. If you had to pick only one among intelligence and hardworking, which one exemplifies you more and why? On a scale of 1 to 10, 1 being the hardest working and 10 being the most intelligent, where would you rate yourself? You can't choose 5.

WHARTON GENERAL BEHAVIORAL
1. Why did you choose Wharton? 2. What other business schools did you get into? 3. Tell me about your activities at Wharton. 4. Why did you choose your campus involvements? What did you gain? 5. How have you contributed to the community at Wharton? 6. What has been your greatest learning at Wharton? 7. What was unexpected, both positively and negatively, at Wharton? 8. What factors did you consider in deciding on your major? 9. What's your proudest accomplishment at Wharton? 10. Why did you not make Director’s List this year? 11. Which subject did you perform best in at Wharton? Worst? Which did you find most interesting (enjoy the most)? Which did you enjoy the least? Which did you find most difficult? Which did you find easiest?
Investment Banking Resource Guide Page 59 of 108

Wharton Finance Club, 2007-2008

INITIATIVE / LEADERSHIP
Bankers see themselves as leaders. They are not the kind of people who just do what they are told or try to meet expectations. Bankers need to take the initiative, to go after clients, and to get internal groups to deliver for their clients. 1. Talk about a leadership experience. Describe a leadership role of yours. Why did you commit your time to it? How did you feel about it? Tell me about a team project when you had to take the lead or take charge of the project? What did you do? How did you do it? What was the result? 2. Define leadership and describe your leadership style. 3. Give me an example of a leadership role you have held when not everything went as planned. 4. Tell me about a time when you were entrepreneurial or took initiative, and what the results were. Tell me about a time when your initiative caused a change to occur. Tell me about a project you initiated. What did you do? Why? What was the outcome? Were you pleased with the result? 5. Describe a situation in which you recognized a potential problem as an opportunity. What did you do? What was the result? What do you wish you had done differently? 6. Tell me about a time you overcame adversity/greatest challenge as a leader. 7. What is the toughest group that you have had to get cooperation from? What were the obstacles? How did you handle the situation? What were the reactions of the group members? What was the end result? 8. Tell me about a time when you successfully resolved a conflict. 9. How would you feel about taking orders from a VP who is younger than you? 10. Describe a situation where others you were working with on a project disagreed with your ideas. What did you do? 11. What are some key lessons you have learned about motivating people? 12. Tell me about a time when you had to make a decision without all the information you needed or under a faster timeline than you would have preferred. What obstacles did you face? How did you handle it? Why? Were you happy with the outcome?
Investment Banking Resource Guide Page 60 of 108

Wharton Finance Club, 2007-2008

13. Describe a recent unpopular decision you made. How was it received? How did you handle it? 14. Give me an example of an important goal, which you have set and tell me how you reached it. What steps did you take? What obstacles did you encounter? How did you overcome the obstacles? 15. Tell me about a goal that you set that you did not reach. What steps did you take? What obstacles did you encounter? How did it make you feel? 16. How competitive are you?

TEAMWORK / INTERPERSONAL
Bankers work in teams and a deeply concerned about the ability of MBAs to work with both their junior and senior peers. They are interested in conflict management, stress management, and client management – all of which fall under this category. 1. Do you like working with people? Do you want that to be part of your job? 2. Give an example of how you worked on a team. What role did you play? 3. How would your learning team describe you? 4. What would cohort mates, other than your Learning Team, say about you? 5. How do you deal with team breakdown/difficulty/disagreement? 6. Tell me about a time when you worked with a classmate or colleague who was not doing their share of work. How did you handle it? 7. Give me 3 examples of situations that you were in a conflict in 30 seconds. 8. Describe a situation in which you had to arrive at a compromise or help others to compromise. What was your role? What steps did you take? What was the result? 9. Describe a situation in which you were able to build team spirit during a time of low morale. 10. Who do you like the least on your learning team and why?

Investment Banking Resource Guide Page 61 of 108

Wharton Finance Club, 2007-2008

11. Give an example of working with someone of another nationality or background. What problems did you have and how did you solve them? 12. What kinds of people do you have trouble getting along with? Why? How do you handle it? How did the relationship progress? 13. Describe a situation where you had a conflict with another individual, and how you dealt with it. What was the outcome? How did you feel about it? 14. What, in your opinion, are the key ingredients in guiding and maintaining successful business relationships? Give examples of how you have made these work for you? 15. Tell me about a time when you had to adjust to a classmate’s or colleague’s working style in order to complete a project or achieve your objectives. 16. Have you ever had to “sell” an idea to your classmates or co-workers? How did you do it? Did they accept your idea? 17. How do you get people to do things they don’t like to do? 18. Give me an example of a time when you were able to successfully communicate with another person even when that individual may not have personally liked you (or vice versa). How did you handle the situation? What obstacles or difficulties did you face? How did you deal with them? 19. Tell me about a time in which you had to use your written communication skills in order to get an important point across.

ADAPTABILITY
The banking lifestyle is notoriously volatile: unpredictable hours combined with constantly changing projects, client needs, and market conditions. These questions are a gut check on whether you are prepared to survive – and thrive – under these conditions. 1. How do you work under pressure and stress? 2. Tell me about a situation in which you have had to adjust to changes over which you had no control. How did you handle it? 3. Describe a situation that required you to do a number of things at the same time. How did you handle it? What was the result? How do you prioritize projects and tasks when scheduling your time? Give me some examples.
Investment Banking Resource Guide Page 62 of 108

Wharton Finance Club, 2007-2008

4. Tell me about a project that you planned. How did you organize and schedule the tasks? Tell me about your action plan. 5. Tell me a time of working under intense time pressure or deadline. How did you handle it and react? 6. Give me an example when you had to work on several different things a time? Did you reach a point where you had to drop one or more things you were working on?

PROBLEM SOLVING
Bankers are fundamentally interested in the ability to “get things done.” These questions help look at your track record in solving problems. Use the opportunity to demonstrate proven competence in transferable skills. 1. Give me an example from your prior job on how you demonstrated your quantitative ability. 2. Describe the project or situation that best demonstrates your analytical abilities. What was your role? 3. Tell me about a time when you had to analyze information and make a recommendation. To whom did you make the recommendation? What was your reasoning? What kind of thought process did you go through? Why? Was the recommendation accepted? If not, why? 4. What steps do you follow to study a problem before making a decision? Why? 5. Tell me about a situation where you had to solve a difficult problem. What did you do? What was your thought process? What was the outcome? What do you wish you had done differently? 6. When was the last time you thought “outside the box” and how did you do it? Why? Tell me about a problem that you’ve solved in a unique or unusual way. What was the outcome? Were you happy or satisfied with it? 7. Tell me about a time someone brought you a new idea that was odd or unusual. What did you do?

Investment Banking Resource Guide Page 63 of 108

Wharton Finance Club, 2007-2008

QUESTIONS FROM THE DARK SIDE
Every year, people report getting questions that are difficult, even dark. Some are more common than others. In preparing responses to negative questions, the key is to transform them into questions you’ve seen elsewhere and to deliver positive responses. Negative feedback in a learning team, for example, becomes “What are your weaknesses?” You know how to handle that. 1. Based on your resume, what do you think concerns us about you? Knowing what you know about yourself, what would concern us most about you? 2. Is there anything that could potentially interfere with your performance? Is there anything about your background that will not make you a good Associate? 3. What were the negative comments on your last review at your previous employer? How were you told to improve your performance? 4. What negative feedback did you get at your learning team feedback session? 5. Of those banks you’ve interviewed with up to 2nd or 3rd round and didn’t give you offer, what feedback did they give you for why? If not, why do you think they didn’t give offers? 6. Why would we not have given you an offer at the end of the Summer? 7. Why would you be dismissed for something that was in your control? 8. Give an example of a past mistake/failure/your greatest disappointment. (Follow up: Give me 3 more.) 9. How did your greatest weakness affect you on your last job? 10. Give an example of a lapse in judgment in your professional life. 11. Describe a time you have had to compromise on your standards and why?

Investment Banking Resource Guide Page 64 of 108

Wharton Finance Club, 2007-2008

BANKING
These questions are intended to demonstrate whether you understand what a career in banking is about. Some focus on your understanding of the role of the Associate and your transferable skills. Others are “situational” questions meant to test your judgment regarding common situations that arise in banking. 1. What do you think are the three most important criteria for hiring someone into this position? What about your background will make you a good investment banker? 2. What do you think are the typical duties of an Associate? 3. If you can relate a sports position on a team to an Associate's role, what would it be? 4. What do you expect to get from your summer experience? 5. Why do you think we selected your resume out of the hundreds that we received for an interview? 6. Why we should hire you instead of your classmates? 7. Tell me something you’d like us to know that’s not on your resume. (Follow up: Tell us another.) 8. You say that you are very passionate and interested in investment banking. Why did it take you so long to make this switch? 9. Would you rather be on the buy-side or sell-side of a transaction? 10. What don’t you like about banking? 11. If you were not offered a position in investment banking, what other jobs would you consider? 12. Where do you see yourself in 5 years? 15? 13. How much do you think you’ll be earning in 10 years? 14. What’s the lowest salary you’d consider? 15. What will you miss most about your personal life due to working in banking?
Investment Banking Resource Guide Page 65 of 108

Wharton Finance Club, 2007-2008

16. Say your manager leaves an assignment for you to complete and then leaves town. You don’t fully understand the assignment and you can’t reach the manager. What do you do? 17. Assuming you get an offer, how will you make your decision between us and other firms? 18. Once you have an offer, how long you think you will need to decide? 19. What is your top concern about recent happenings in the investment banking industry? 20. If two clients approaching you, both want to bid for the same target, which client would you choose to represent and why? 21. If you had four voice messages from various MDs for projects, how would you prioritize them? 22. You're the Associate who's put together the presentation and done all the analysis. You're on your way to a client meeting. When you get there, your MD and VP are nowhere to be found and can't be reached. What do you do? 23. Your MD is making a terrible suggestion to a client at a client meeting. What do you do? 24. What would you say on the phone to the client that you are not representing? When they answer, if you don't represent me, I am going to other banks, how do you respond? 25. If you are working for an MD's pitch to be delivered tomorrow and another MD calls you and asks you to work on a new pitch by tomorrow morning, what would you tell him? 26. Imagine that you are the Associate in charge of a project and you are working at the client's office. The CEO of the client company comes to you and complains that the Managing Director does not spend enough time with them. How would you react? 27. If you are selling a company and a prospective buyer calls and asks how many other people there are submitting bids on the company, what do you say? 28. Would you have a problem being told to make photocopies?

Investment Banking Resource Guide Page 66 of 108

Wharton Finance Club, 2007-2008

8.4

Technical / Quantitative Questions

Wharton students love to prepare technical questions. In fact, there’s a 50% chance you didn’t even really read the behavioral questions just so you could get to these questions instead. Here’s the problem with preparing for technical questions: banks frequently tell Career Management that all Wharton students sound the same. Admittedly, this guide is part of the problem. But the deeper issue is people taking the answers to technical questions and memorizing them verbatim. Think about it for a minute before doing this: How do you expect to gain competitive advantage by commoditizing your answers? Nonetheless, technical questions are an important part of the interview and should be prepared for. If you don’t know the answer to the most basic technical questions, you will have some uncomfortable interview moments in which you look foolish. But remember, technical questions are a threshold test. They will not ultimately get you the job. The behavioral part of the interview – you demonstrated interest in banking and ability to succeed in it – are much more important. Before worrying about the answer to every possible technical question, make sure that your fundamentals are firmly in place. Use this section to develop the necessary mastery: but notice that there are many other questions in the behavioral sections earlier in this guide that will be more important in determining the outcome of your interviewing. Answering Technical Questions The most important point when answering conceptual/quantitative/technical questions is to voice your thinking. Similar to an exam, an interviewer is likely to give you partial credit (if not significant credit) if you don’t answer correctly but if they can listen to your thinking and understand how you dissect problems. It is best to try and provide some sort of answer to a question and, as stated previously, articulating your thinking (even if you are unsure of the exact answer) will always help you. The balance to this advice is to ensure you do not ramble with your answer in an effort to try and talk your way out of delivering a direct answer. This will not work and you are likely to only aggravate the interviewer. However, if you genuinely do not know the answer to a question, seek further clarification or more information that may help you. If you still don’t know, state that you do not know the specific answer to the question and then provide a brief response that looks to address some part of the question. It is OK to take a few moments to compose your thoughts but don't take too long or stare blankly at the interviewer.
Investment Banking Resource Guide Page 67 of 108

Wharton Finance Club, 2007-2008

GENERAL FINANCE AND BANKING
1. Conceptually, how does a firm create value? • A firm creates value by generating return on invested capital, growth, and positive cash flows. A firm should seek to earn a return on its invested capital greater than the opportunity cost of capital. By growing (as long as its return still exceeds cost of capital), the firm creates more value. In doing so, the firm generates cash flows (or economic profits).

2. What is a P/E (Price/Earnings) ratio and why do analysts use it? What quantitative and qualitative factors drives the P/E multiple? Explain the difference between two companies with identical earnings but different multiples. • • P/E Ratio (or multiple) = Price of Stock / Earnings per Share The P/E multiple is used in comparing the relative attractiveness of stocks. It gives investors an idea of how much the market is paying for a company’s earning power. For each dollar of stock price, a certain amount of earnings is expected. The higher the P/E, the more investors are paying, and therefore the more earnings growth they are expecting. A P/E multiple above 20x generally implies high growth. The P/E of the S&P 500 is 16-17x. Quantitatively, P/E is moved by changes in share price and earnings through the numerator and denominator, respectively. Qualitatively, share price is affected by market perceptions/expectations of risk, growth, quality of earnings (margins), and general investor confidence.

• •

3. What do you get when you multiply a firm's Net Income by its P/E ratio? • Market capitalization.

4. What is EBIT? • EBIT refers to earnings before interest and taxes. EBIT is an approximate measure of operating income and a common measure of operating comparability. Because EBIT doesn’t include interest or taxes, it provides a measure of income independent of the firm’s capital structure.

Investment Banking Resource Guide Page 68 of 108

Wharton Finance Club, 2007-2008

5. What is EBITDA? • EBITDA refers to earnings before interest, taxes, depreciation and appreciation. EBITDA is an approximate measure of free cash flow – but it does not measure actual free cash flow. By excluding interest and taxes (effects of capital structure) as well as depreciation and amortization (noncash expenses), EBITDA gives a sense of a company’s ability to turn profits. EBITDA is widely used in financial analysis and valuation. For example, interest coverage ratio (EBITDA/interest expense) represents a company’s ability to pay interest to its creditors. Debt service ratio (EBITDA/Outstanding balance of long term loan) measures a company’s ability to honor its long-term debt. EBITDA excludes capital expenditures, working capital needs, dividends, and principal repayments on debt. This can lead to distortions in different industries, for example telecom.





6. What is the Capital Asset Pricing Model? • CAPM is a model designed to find the expected return on an investment and therefore the appropriate discount rate for a company’s cash flows. It is a linear model with one independent variable, Beta. CAPM divides the risk of holding risky assets into systematic and specific risk. To the extent that any asset is affected by general market moves, that asset entails systematic risk. Specific risk is the risk which is unique to an individual asset. It represents the component of an asset's volatility which is uncorrelated with general market moves. According to CAPM, the marketplace compensates investors for taking systematic risk, but not for taking specific risk. CAPM considers a simplified world where there are no taxes or transaction costs. All investors have identical investment horizons. All investors have identical perceptions regarding the expected returns, volatilities and correlations of available risky investments.





7. What is the difference between Equity and Enterprise Value? • • Equity value (market value, market capitalization) is the stock price of a firm multiplied by the shares outstanding. Enterprise Value (aggregate value, total value) includes the market value of the equity and the value of debt (net of cash).
Investment Banking Resource Guide Page 69 of 108

Wharton Finance Club, 2007-2008



For example, a firm with 100,000 shares outstanding, a stock price of $20, debt of $500,000 and $500,000 in cash has an enterprise value of $2,300,000. (Enterprise value also includes preferred stock and minority interest.) Enterprise value reflects the discounted future cash flows to all claimants, whereas equity value reflects the discounted future cash flows only to equity holders.



8. Would you use Enterprise Value/Net Income as a multiple? • No. When using earnings or revenue related ratios, total value to investor base (numerator) should be matched with potential cash available to them (denominator). Hence, enterprise value, which includes both debt and equity, should be taken as a multiple of earnings before interest is extracted. Similarly, when calculating equity/market value ratios, denominator should be earnings available to equity holders, i.e. earnings after interests have been extracted.

9. If one of your clients had extra cash, how would you tell them to invest it? • As a general rule, managers should do one of two things with excess cash: invest them in positive NPV projects (including acquisitions, capital expenditures, and research and development) or return the money to stakeholders in the form of share repurchases, dividends, and debt repayments. Considerations include the cost of debt, the cost of equity, deviation from optimal WACC levels, and tax considerations. What constitutes “excess” cash is a subjective question. In cyclical industries, for example, paying out large amounts of cash may leave the company unprepared for a subsequent downturn.



10. What are the two methods of valuing a traditional option? What factors are considered? What is the difference between a put and a call? • • The two methods are Black-Scholes and the binomial model. The inputs are: o Current Share Price o Exercise Price o Time To Maturity o Risk Free Rate o Variance of Return on the Stock A put is an option to buy, a call is an option to sell.
Investment Banking Resource Guide Page 70 of 108



Wharton Finance Club, 2007-2008

11. In a perfect (tax free) world, if you have a company with an enterprise value of $5 billion and you take out $2 billion in debt, what is the new enterprise value? What is the enterprise value if you subsequently use the $2 billion to pay out a dividend? What is the enterprise value if instead of paying out the dividend you invest the $2 billion in a new project with an NPV of $3 billion? • Issuing the $2 billion in debt increases cash by $2 billion but also increases debt by $2 billion. Cash and debt net out. Thus firm value remains the same at $5 billion. Paying out a dividend of $2 billion eliminates cash on the balance sheet and, in doing so, eliminates $2 billion in equity value. Thus firm value remains the same at $5 billion. Using the $2 billion to invest in a project with an NPV of $3 billion increases the enterprise value of the firm by the investment amount ($2 billion) and the investment’s NPV ($3 billion). Thus firm value is $10 billion.





12. What is the difference between common stock and preferred stock? How do they trade relative to each other? • Preferred stock, similar to debt, is guaranteed a dividend. Common stock is not. Preferred stock also has a priority over the company’s assets versus common stock. As a result, preferred stock trades at a premium.

13. What is the calculation for EPS? Does that include preferred stock? What about convertible bonds? • Net Earnings-Per-Share (EPS) is the portion of a company's net earnings allocated to each share of stock. It is calculated by dividing net earnings by common shares outstanding adjusted for the assumed conversion of all potentially dilutive securities. EPS does not include preferred stock. Securities having a dilutive effect may include convertible debentures, warrants, options, and convertible preferred stock.

• •

14. How do you calculate fully diluted shares in order to calculate fully diluted EPS? • Use the Treasury Stock Method.
Investment Banking Resource Guide Page 71 of 108

Wharton Finance Club, 2007-2008

15. What happens to EPS when a company repurchases its stock? • All else remaining the same (i.e. earnings level stays same, debt level is not increased, etc.), EPS will increase as total number of shares outstanding decreases. If debt had been issued to repurchase stock, EPS change depends on interest expense relative to net income, vs. shares repurchased relative to shares outstanding. If a company with EPS of $1.00 pays all cash to buy another company with EPS of $0.50, why is the EPS of the combined company not $1.50? The earnings can be added, but the shares cannot. The exchange ratio of the shares will determine the denominator and EPS could be $1.50, or accretive or dilutive.

• •

16. What is Cash EPS? Why is it used on some industries? • Cash EPS is a measure of financial performance that looks at the cash flow generated by a company on a per share basis. The higher a company’s cash EPS, the better it is considered to have performed over the period. Cash EPS = Operating Cash Flow / Diluted Shares Outstanding Cash EPS is used in industries that rely heavily on stock options (like tech) because of SEC requirements that firms expense stock options. As a result, Net Income is decreased by a non-cash expense which is adjusted under Cash EPS.

• •

17. What is an interest coverage ratio? What is a leverage ratio? What do you use them for? 18. Why might a company be trading at a lower EV/EBITDA multiple than its competitors of the same size in the same industry? 19. If you have a company with negative EBITDA but positive gross margin, how would you value that company? 20. How would you adjust the enterprise value for a company with significant pension liabilities? Would the firm's new EV/EBITDA multiple be comparable to other firms? What adjustments would you make to EBITDA to create a comp?

Investment Banking Resource Guide Page 72 of 108

Wharton Finance Club, 2007-2008

21. What metrics can you use to measure a company’s risk? It’s growth? It’s profitability? 22. Walk me through your favorite chapter of your finance book. 23. In a world with taxes, if you issue debt for $100 and pay it out as a dividend how does it affect your enterprise value?

VALUATION
1. How would you value a company? • There are five common valuation methodologies. o Trading Range (the range the stock has been trading in during the past 52-weeks). If we trust the market we should assume this is a reasonable place to start our analysis. Discounted Cash Flow (DCF), also called Intrinsic Value, seeks to find a present value of all future cash flows of the firm that are available to stakeholders. This can be done using WACC or APV. Public Comparables looks at peer companies to determine how the market values companies in the same or similar businesses using multiples including P/E and EV/EBITDA. Acquisition Comparables, also called Deal Comps or Precedent Transactions, looks to see how much acquirers recently paid for similar businesses. Asset Value, also called liquidation or break up value, examines what you can sell the company’s assets for (including real estate).

o

o

o

o •

In addition, valuation can be framed through: o Leveraged Buyout looks at what a financial sponsor could pay considering a target IRR and the debt capacity of the firm. Merger Consequences Analysis is actually an affordability analysis (what can an acquirer pay) rather than an analysis of the value of a target.

o

Investment Banking Resource Guide Page 73 of 108

Wharton Finance Club, 2007-2008

2. Which of the valuation methods will tend to lead to the highest valuation? • Acquisition Comparables tends to lead to the highest valuations because it includes not only the stand alone value of the business but also synergies expected in the transaction as well as any premium paid for achieving control. In recent times, LBO has risen higher in the ranks of valuation analysis, although previously it was low on the scale because undervalued companies were often the targets. Public Comparables will tend to lead to the lowest valuations because they involve the markets expectations. DCF, on the other hand, will factor in management’s sometimes optimistic forecasts for future growth. In fact, some would put DCF as the highest valuation if management has particularly rosy scenarios and includes synergies.





3. Which method of valuation is most robust? What are the pluses/minuses of each method? • Discounted Cash Flow is the theoretical intrinsic value of the firm: looking at the actual, unlevered cash flows. It is an objective calculation that is always attainable. It takes into account the synergies and tax benefits of the capital structure and shows the maximum an acquirer should be willing to pay. DCF is limited by the extreme subjectivity of its inputs. There is high sensitivity to changes in growth and profitability forecasts, as well as discount rates. Another limitation is that most DCFs in banking are done with WACC, which doesn’t allow for varying debt levels and costs. Public Comparables have as their advantage that they focus on companies with similar operations and financial situations allowing for relative, market oriented comparisons. Their disadvantages are that they rely on publicly available information, are subject to changes in market conditions, and are often not “pure” comparables because of differences in businesses even in the same industry. Acquisition Comparables requires identifying similar transactions or firms and then using various ratios to see how much the target firm should be worth. Acquisition Comparables have the same drawbacks as Public Comparables exacerbated by the possible lack of comparability in synergies and control premia. Market conditions at the time of the transaction and any special conditions should be taken into consideration. Leveraged Buyout is essentially a DCF with special conditions.
Investment Banking Resource Guide Page 74 of 108







Wharton Finance Club, 2007-2008

4. What is unlevered free cash flow (FCF)? • FCF represents the cash that a company is able to generate after laying out the money required to maintain or expand its asset base. FCF is important because it allows a company to pursue opportunities that enhance shareholder value. Without cash, it’s tough to develop new products, make acquisitions, pay dividends, and reduce debt.

5. Why does DCF used unlevered cash flow? • DCF uses unlevered cash flows because these are cash flows that are generated by the firm’s real assets. Financing is considered in subsequent steps when discounting using either WACC or using the APV method. Therefore if we were NOT using unlevered cash flows, when discounting these cashflows using WACC we would be “double-counting” the refinancing effect.

6. How do you calculate unlevered free cash flow (FCF) starting from Net Income? • Net income + Interest expense*(1-t) + depreciation - capex - investment in working capital = FCF to firm - interest expense*(1-t) - principal repayments = FCF to equity

7. How do you calculate unlevered free cash flow (FCF) starting from EBITDA? • EBITDA*(1-t) + depreciation*t - capex - investment in working capital = FCF to firm - interest - principal repayments = FCF to equity

8. What is a Terminal Value? How do you calculate it? • • Terminal value (TV) refers to the value at the end of a given projection period. Terminal Value can be calculated in four ways (the first two are by far the most common): o Perpetuity Growth, also called the Gordon Growth model, takes the last year’s normalized net cash flow in the terminal year, multiplies it by 1+growth rate and then divides by the discount rate minus the perpetual growth rate. (CF*(1+g)/(k-g)). Perpetuity Growth is, of course, highly sensitive to its input assumptions. The perpetual growth is usually calculated as being somewhere between inflation and GDP growth (2-5%).
Investment Banking Resource Guide Page 75 of 108

Wharton Finance Club, 2007-2008

o

EBIT or EBITDA multiple. Assumes that the perpetual value will be in line with the multiple of the terminal year, or often even the current year. Using multiples takes out the uncertainty of determining future growth rates. Additionally, multiples are more relevant when anticipating sale of company to public markets – they project investors’ willingness to pay relative to earnings. They must be used with particular care in cyclical industries. P/E multiple of the net income of the terminal year, assuming the company will be worth the same multiple of its future earnings going forward. This approach is complicated by the difficulty of determining the appropriate P/E in the future. Liquidation Value in the terminal year.

o

o

9. What percent of total DCF value is usually in the Terminal Value? What proportion did the Terminal Value contribute to the Enterprise Value in your Time Warner Valuation. Why? What concerns are there? • The amount of the enterprise PV in the terminal value depends on the length of the forecast period and the growth rate of the cash flows. The longer the forecast period, the smaller the percentage in the terminal value. At the same time, the more growth that is assumed, the farther the value is pushed into the future. Terminal value is usually the majority of the total value, and often threequarters of the value. The larger the terminal value becomes as a percentage of total value, the more sensitive a DCF valuation is to its assumptions, hence the difficult in using it with high growth firms.



10. What is WACC? How would you calculate it? • • WACC is the Weighted Average Cost of Capital. It is the tax-affected, capital structure weighted, opportunity cost of capital. WACC is used as the discount rate for assigning present values to future, unlevered free cash flows to all stakeholders. In capital budgeting, it is used to find the NPV for a project. WACC = Debt/(Debt+Equity) * Return on Debt * (1-tax rate)+ Equity/(Debt+Equity)*Return on Equity Return on Debt = Yields implied by the trading price of a company's outstanding debt
Investment Banking Resource Guide Page 76 of 108



Wharton Finance Club, 2007-2008

Return on Equity = Risk Free Rate + Beta*(Market Premium - Risk Free Rate) Capital Structure = Debt/(Debt+Equity): Since capital structures change over time, it is usually not based on a company's current structure, but rather some target capital structure, often derived from examination of the industry average. 11. Why do you have a (1-T) in WACC? • Interest paid on a firm’s borrowings can be deducted from taxable income (the interest tax shield) and must be accounted for.

12. What is Beta? How and why would you “unlever” it? • Beta is a measure of systematic risk: the sensitivity of a security's return in relation to the market. If a stock has a beta of more than 1, it means that it is more volatile, or “swings” more than the market. If a stock has a beta of 1, it moves in unison with the market (an S&P 500 index fund, for example, move close to identically as the market as a whole). If the beta is less than 1, the stock moves less than the market. For example, a portfolio which has a beta of 0.5 will tend to participate in broad market moves, but only half as much as the market overall. Stocks with higher betas have higher risk and also tend to have higher returns over time. Basically, a stable blue-chip would have a beta of close to 1, where a semiconductor firm might have a beta of more than 1. For firms or divisions that are not publicly traded, beta can be estimated from the betas of comparable firms. In order to account for differences in leverage, however, the betas of the comparable firms need to be unlevered, averaged, and then relevered using the capital structure of the firm the beta is being estimated for. Beta can be unlevered using the following equation: BU = BL / [1 + (1-t)*(D/E)] And relevered using this one: BL = BU (1 + (1-t)*(D/E)]





• •

Investment Banking Resource Guide Page 77 of 108

Wharton Finance Club, 2007-2008

13. You are an Associate and I am you new Analyst. Walk me through a DCF valuation. How do you estimate costs of financial distress? • Step 1. Look at the financial statements to obtain information regarding EBITDA, depreciation, capital expenditures (capex), changes in net working capital (NWC), book value of debt and equity, etc. Step 2. Determine the future free cash flow. Free cash flow (FCF) is the net cash flow available to stakeholders after paying for future investments. o FCF can be calculated as EBITDA – D&A – taxes + depreciation – Cap Ex - increase in net working capital. (To get to FCF just for equity holders, also subtract interest and principal repayments.) FCF can also be calculated as Net Income + (interest expense * 1-t) + D&A – Cap Ex – increase in net working capital. (To get to FCF just for equity holders, also subtract interest expense (times 1-t) and principal repayments.



o



Step 3. Determine the discount rate. The discount rate is determined differently in the two ways to calculate the DCF: the Adjusted Present Value (APV) method and the Weighted Average Cost of Capital (WACC) method. The difference lies on how the tax shield effect is accounted for when calculating NPV. Because interest expense is tax deductable, companies gain some benefit by issuing debt for their financing needs. WACC accounts for this benefit directly in the discount rate while APV accounts for this benefit in the cash flow calculations while holding the discount rate the same as an all-equity financed firm. Taking into account the tax shield, the WACC is calculated as:



WACC = rD (1 − TC )

D E + rE V V

where rd is the required return on debt and re is the required return on equity. D, E and V are the values of debt, equity and the total value of the firm, respectively. Tc is the corporate tax rate. • • In the APV method, the discount rate will just be the rate for the all-equity financed firm, which can be calculated using CAPM. Step 4. Determine the terminal value. When we assume that the company is an ongoing concern, we need to make an assumption about all future cash flows beyond the point we have data. We therefore can take the last time
Investment Banking Resource Guide Page 78 of 108

Wharton Finance Club, 2007-2008

period’s cash flow the last year for which we have data) and assume an annual average growth rate for the cash flow. • The terminal value in that time period is therefore

FCFT (1 + g ) r−g
• where g is the annual average growth rate of cash flows, and r is the company’s discount rate. In the WACC method, it is the WACC. In the APV method, it is the unadjusted firm discount rate. Step 5. Calculate the company’s net present value. Using the WACC method, we can discount the FCFs and the terminal value by the calculated WACC to obtain the NPV. Using the APV method, we need to add the PV of change in the financing decisions to the NPV of an allequity firm. We therefore need to add the tax shield to the cash flows and terminal value and discount them using the discount rate for the all-equity firm. Financial distress occurs when a company has difficulty or is unable to pay back loans/interest. Financial distress is costly. Investors know that levered companies may fall into financial distress and they worry about it. That worry is reflected in the current market value of the levered company’s securities. Thus the value of the company can be broken down into three parts: Value of company = value if all-equity-financed + PV(tax shield) – PV(costs of financial distress) Therefore the cost of financial distress can be estimated from the difference between the sum of the value of an all equity company and the PV of tax shield and the current value of the company.

• •



• •

14. How do you perform a public comparable valuation? How do you choose a comparable peer group? • Identify peer companies. You would do this by looking at companies with similar operations (products/services, customers/clients, distribution, and geography) and financial aspects (size). You would get this information from the following sources:

Investment Banking Resource Guide Page 79 of 108

Wharton Finance Club, 2007-2008

o o o o •

Previous analyses of other bankers – public filings (especially fairness opinions) Peer group index (proxy statement), competition section (10-K or IPO prospectus) Research, S&P, Value Line, Bloomberg Industrial classification screen from database (Capital IQ)

Good comparables have similar operational and financial aspects. That is, they are comparable in terms of size, risk, and growth. In particular, they are from the same or similar businesses, are in the same size range (market cap, revenues), have similar growth expectations, and similar capital structures. Once you have identified your peer group then you need to get their financials: o o o o 10K or annual report from latest fiscal year 10Q from latest quarter (or more recent press release with financial details) Research and EPS estimates (Thomson Financial Fist Call, Reuters Estimates(Multex) Share price (and latest dividends)



15. What are some common ratios used to compare equity performance? • • • • Price / EPS Market Value / Net Income Market Value / Book Value Price to Earnings / Growth Rate (“PEG Ratio”) – applying growth to a multiple

16. What are some common ratios used to compare enterprise performance? • • • EV / EBITDA EV / EBIT EV / Sales (generally only appropriate for volume driven businesses or those with negative earnings)

17. Discuss APV vs. WACC. • WACC and APV are slightly different valuation methodologies for doing a DCF. Wall Street essentially only uses WACC.

Investment Banking Resource Guide Page 80 of 108

Wharton Finance Club, 2007-2008



WACC is based on a firm's current capital structure. It explicitly adjusts for the capital structure by incorporating tax shields into the discount rate, which can be seen clearly in the WACC formula. WACC's limitation is that it gives less accurate results when the business risk and debt ratio are expected to change. APV looks at tax shields separately from other effects. This allows risk and debt to change over time by treating financing effects separately. This is important for highly leveraged, complex, or changing capital structures. The APV discount rate is calculated by using the CAPM to determine the return on equity based on the firm’s unlevered beta.



18. How do you value a private company? • Valuation for a private company is the same as the valuation of a public company with some complications, particularly as it relates to DCF. Because a private company has no publicly traded equity, a beta cannot be directly computed. To find Re: o o o o o • Estimate the total value of the private company based on comparables (use average EV/EBITDA). Deduct the value of debt to get estimated "market" value of equity. Get the average levered beta from the comparables and unlever it. Relever the beta for the private company based on target D/E. Calculate Re based on CAPM.



To find Rd: o o o Some private companies have publicly traded debt - so look up trading yields to estimate Rd. Alternatively, estimate what the credit rating of the private company would be based on comparables (look at credit statistics). For estimated credit rating, use current market yields for similarly rated companies to determine Rd.



Calculate WACC as normal.

19. Why are some stock options relevant to valuation? • Unexercised, in-the-money options represent implicit equity value not reflected in the current market cap but that should be included.
Investment Banking Resource Guide Page 81 of 108

Wharton Finance Club, 2007-2008

20. Explain the difference between WACC and IRR. 21. For a stable, mature company, if you use an EBITDA multiple to calculate Terminal Value, will the multiple be high or low? 22. What is the beta of a lottery ticket? 23. You are sitting with a CEO and can only ask him three questions about his business before giving an estimate of its value. What questions would you ask? 24. Walk me through how would you would calculate your Present Value. What is your beta? What rate would you use to discount yourself? Give me an idea of the projections for your cash flows. 25. What levers on a DCF can be manipulated to change its outcome? 26. How would you value an asset management company? 27. How would you value a supermarket? 28. How do you value a diversified industrial conglomerate? 29. How would you find the appropriate discount rate for an internet based retailer? 30. Of the three valuation methods, which one is most appropriate for a strip mall company? A biotech firm? 31. How do you treat deferred taxes in a DCF? 32. How would you find the cost of equity for an Indonesian company? What drives the cost of equity rate? 33. You have a cable company and a law firm; describe some of the differences between them, particularly with respect to valuation and capital structure.

MERGERS AND ACQUISITIONS
1. Why would two companies merge? What major factors drive mergers and acquisitions? • Mergers are driven by three main factors: revenue synergies, cost synergies, and strategic concerns.
Investment Banking Resource Guide Page 82 of 108

Wharton Finance Club, 2007-2008

o

Revenue synergies include gaining access to new markets for existing products, broadening of product lines, gaining market share in the same market, cross selling products to customers, locking in customers through vertical integration, creating network effects, or enhancing brand recognition. Cost synergies include economies of scale and scope and reduced overhead costs like administration, IT, and rent. Strategic concerns include preventing competitors from entering a market, defending against being acquired, and diversification.

o

o

2. What are the criteria to consider when advising a client on whether to make an acquisition or not? • The most fundamental consideration is whether the transaction increases shareholder value (NPV > 0). Primary considerations are financial and strategic. Financial considerations include EPS accretion/dilution, ability to finance the acquisition, impact on margins, tax implications, and cost of capital concerns. Strategic considerations include growth in market share, industry trends, vertical integration, leadership and management concerns, technology, and the regulatory and political environment.





3. What is the typical M&A transaction process? • • • Assess goals & objective of M&A Consider impact on acquirer: valuation, deal structure, financing, tax implications, post-merger integration Evaluate impact on regulatory & competitive environment

34. Company A wants to buy Company B for $400m cash, the maximum they think it is worth. Under what circumstances might Company A agree to paying $430m in a stock transaction rather than a cash one? • Alternative 1: If Company A and B agree to convert the deal from cash deal to stock deal and Company A and B have different perspectives on the value of Company A: Company A expects that the post-merger value of Company A would be lower than Company B does.
Investment Banking Resource Guide Page 83 of 108

Wharton Finance Club, 2007-2008

• •

Alternative 2: Company A uncovers more additional synergies. Alternative 3: Company A faces unexpected increase in profit and expects that Company B would record in loss. So the merger would bring more than $30m in tax benefits to Company A. For example, Company A has $400 m in earnings before tax and Company B has a $200m loss in earnings before tax then Company would get $70m in tax benefits (assuming a tax rate of 35%). Alternative 4: If Company A and B agree to convert deal from cash deal to stock deal and shareholders of Company B expects to get capital gains by selling Company B, the Company B shareholders’ marginal tax rate would change from 39.6% (ordinary income marginal tax rate) to 20% (capital gains tax rate for the selling stock at the market). So there is a room for further negotiation.



4. What is a fairness opinion? Why are past prices important in compiling one? • A fairness opinion is a professional opinion issued by an investment bank used for guidance in a merger, takeover, or acquisition. Past prices of relevant companies in similar transactions give you a benchmark to determine fair value for a company.

5. What are some common hostile takeover defense tactics? • • • • • • • Massive share repurchase (spend cash, decrease available shares) Poison Pill (allows target to issue new shares that can be redeemed at a premium after the takeover thus diluting the shares of the acquirer.) White knight (find another buyer) Leverage buyout Restrictive charter Tiered board election Greenmail (buy shares back from acquirer at a premium – not preferred)

6. If I have a company with three divisions whose stock price is depressed because of the underperformance of one of the divisions, what are five things I could do to improve the stock price? • • • • • Spin off the division Reduce costs of that division Shut down the division Restructure the company Modify accounting methods for the third division (i.e. alter depreciation schedule or inventory recognition method – not preferred)
Investment Banking Resource Guide Page 84 of 108

Wharton Finance Club, 2007-2008

7. What are the pros/cons on a stock vs. cash acquisition? • There are three ways in which a firm can use equity in a transaction. The first is to use cash balances that have been built up over time to finance the acquisition. The second is to issue stock to the public, raise cash and use the cash to pay for the acquisition. The third is to offer stock as payment for the target firm, where the payment is structured in terms of a stock swap – shares in the acquiring firm in exchange for shares in the target firm. A variety of factors need to be considered including competing bidders, taxes, the relative pricing of the securities, dilution, and regulatory issues. The question of which of these approaches is best utilized by a firm cannot be answered without looking at the following factors: o The availability of cash on hand: Clearly, the option of using cash on hand is available only to those firms that have accumulated substantial amounts of cash. Large cash deals also raise financing and capital structure issues (i.e., too much debt vis-a-vis equity). The perceived value of the stock: When stock is issued to the public to raise new funds or when it is offered as payment on acquisitions, the acquiring firm’s managers are making a judgment about what the perceived value of the stock is. In other words, managers who believe that their stock is trading at a price significantly below value should not use stock as currency on acquisitions, since what they gain on the acquisitions can be more than what’s lost in the stock issue. On the other hand, firms that believe their stocks are overvalued are much more likely to use stock as currency in transactions. The stockholders in the target firm are also aware of this, and may demand a larger premium when the payment is made entirely in the form of the acquiring firm’s stock. Stock deals are tax advantageous to sellers because capital gains taxes are deferred until the acquired shares are sold.

• •

o

o

8. What are the pros and cons of a purchase of a target’s assets as opposed to a target’s equity? • • There are two key issues: taxes and risk. A stock purchase is preferred by sellers because the buyer pays the taxes in the future on the assets. The seller also transfers all real and contingent risk to the buyer.
Investment Banking Resource Guide Page 85 of 108

Wharton Finance Club, 2007-2008



An asset purchase is more favorable to the buyer for tax purposes because it is taxable for the seller but deductible for the buyer because of depreciation. It also allows the buyer to pick and choose what it wants to acquire, including avoiding some liabilities (such as environmental damages or pending litigation) to minimize risk.

9. Who would pay more to acquire a company - a financial buyer or a strategic buyer? Why? • Strategic buyers are often willing and able to pay more for a company than financial buyers. Strategic buyers may be able to realize synergistic benefits almost immediately due to economies of scale that may exist through the combined purchasing power of the new entity and the elimination of duplicate functions. The better the fit (i.e., the more realizable the synergies are), the more they will want the business and the greater the premium they will pay.

10. What advantages do financial buyers have? • PE firms have greater ability to take on debt because their cost of debt is lower. Because PE firms seek to maintain management in place, they often create incentives for insiders to favor them over strategic buyers.

11. If a company with higher P/E multiple acquires a company with lower P/E multiples, is the transaction accretive or dilutive? • If a company with a higher P/E acquires a company with a lower P/E the transaction is dilutive. That is to say, the acquisition decreases the acquiring company’s EPS because the price paid by the acquiring firm exceeds the addition to EPS. Dilution can be prevented, or at least minimized, by issuing debt or using cash to purchase the company.



12. In calculating merger consequences, what factors impact the new company’s EPS? • In the numerator, pro forma earnings are a function of acquirer’s net income, target’s net income, synergies, interest on liquidated debt minus interest on new debt, preferred dividends paid out to target shareholders, and goodwill amortization. In the denominator, pro forma shares are a function of acquirer’s existing shares plus new shares issued in the transaction.
Investment Banking Resource Guide Page 86 of 108



Wharton Finance Club, 2007-2008

13. How do you calculate a breakeven offer price under a 100% stock transaction? • Multiply the acquirer’s P/E ratio by the target’s EPS.

14. Why do accretive mergers still sometimes see a falling stock price? • There can be a variety of reasons including multiple contraction, the impact of leverage, or new business risks.

15. Name three alternatives for a company to consider as alternatives to a merger or acquisition. • • • Outsourcing Joint Venture Corporate Restructuring

16. What’s likely to yield greater synergies, a diversification acquisition or a consolidation acquisition? 17. Talk to me about some hostile deals that have been initiated lately. When does a hostile deal make sense? What percent of hostile deals are completed by the buyers that started the process? 18. If your client is planning on selling his company one year from now, what actions would you recommend he take over the next 12 months to increase the price he receives for his firm?

CAPITAL STRUCTURE AND FINANCING
1. Why issue equity versus debt? • • Growth firms often issue equity while traditional firms issue debt. Companies with predominantly tangible assets are more likely to issue more debt than companies with predominantly intangible assets, because of the lower bankruptcy cost. Debt: When a company has sufficient earnings to utilize the tax shield over the life of the debt, the company will choose to issue debt for the tax deduction. The interest payments, however, can be a problem for a new company without stable cash flow.
Investment Banking Resource Guide Page 87 of 108



Wharton Finance Club, 2007-2008



Equity: If highly leveraged, no one will buy debt. In good markets, equity can be issued at a premium. It is a currency for acquisition and does not legally require payouts to holders. It is, however, dilutive for other owners and has a higher cost than debt. Market Signaling: Academic studies have found that the announcement of equity issuing tends to drive down the stock price. Under the assumption that managers in the company have more information than investors. When managers expect to have higher earnings in the future, issuing debt is a better choice due to coverage ratio, tax benefit and financial flexibility. Conversely, if the manager expects that earnings will go down, issuing equity is a better choice since the low coverage ratio and inability to take the advantage of tax benefit. EPS (earnings per share) is a good measure when choosing between debt or equity financing. Although debt financing saves taxes, newly created interest expenses reduce net income. For equity financing, although there is no interest expense, there is possible dividend expense. More importantly, due to the increase in total number of shares outstanding, equity financing has a dilutive effect on earning.





2. What are the three traditional reasons for doing an IPO? • • • Capital constrained (need the money to keep running businesses) Opportunity constrained (need the money to enter new markets) People constrained (need the money to keep / attract talent)

3. What issues should be examined when considering an IPO? • • • • • • • Valuation Accretion / Dilution (EPS) Ownership Coverage ratios Trading comparables Market timing SEC issues

4. If you are pitching to be an underwriter for an IPO, what would the table of contents of the pitch book look like? • • • Introduction / Executive Summary Industry overview (trends) Company positioning, performance
Investment Banking Resource Guide Page 88 of 108

Wharton Finance Club, 2007-2008

• • •

Preliminary valuation considerations – precedent transactions Comparables Overview of your firm / biographies

summary

financial

overview,

5. What is the transaction process of a typical IPO? • Beauty Contest o 7% Fee, bank may undercut o I-banks trying to convince client that they can raise a specific amount of money o Client chooses bank(s) (Book runner, ECM keep track of shares allocated) to lead, then lead bank chooses syndicates (help sell offer) Due diligence/Valuation o Management o Financials o Market SEC Filing (S-1) o Prospectus o Announcement of intent to do IPO o Work with lawyers and syndicates Roadshow o Sell securities to institutional investors o Research goes on separate road show to institutional investors o Offering date







6. Why are IPOs generally underpriced? • There are two general reasons that are given to explain the underpricing of IPOs. The first is conservatism: issuers want their stock to go up on the first day and thus leave a good taste in the mouth of investors. The second is compensation to the initial owners for the risk they are taking, relative to subsequent owners, for buying first.

7. Why does a secondary offering of shares tend to decrease the stock price? • Issuing new shares signals that the management believes that the stock is overvalued. If the company believed that it needed the cash for a successful project, it would have issued debt, thus keeping all of the upside
Investment Banking Resource Guide Page 89 of 108

Wharton Finance Club, 2007-2008

within the existing shareholder base rather than expanding (and diluting) it by adding new shareholders. • A firm might still prefer to issue equity instead of debt if management believes the stock is overvalued, if predictable cash flows are enough of a concern that the firm does not want to risk adding debt, or to pay down debt (maybe to improve a credit rating).

8. If a company has a market cap of $900 million and issues $100 million in new equity, what percent of the company do the new equity owners own? • 10%. 900+100 = 1000. 100/1000 = 10%.

9. What implications are there for cash dividend versus stock repurchase? Why and when would you use one versus the other? • The main advantage of stock repurchase over dividends is that the former is a one-time distribution. An increase in dividends is promised to the shareholders, such that the company would be paying the dividend on a regular basis in the future. There are two important implications of "expected regular dividend" and "one-time distribution." One is that if a regular dividend is lowered, then investors interpret this act as a bad sign, and thus the price of the stock tends to fall. Because of this built-in expectation, dividend is a burden on the company, as the company feels obligated to satisfy its shareholders’ expectations. On the other hand, stock repurchase is a one-time deal, and thus, shareholders don’t expect it to continue in any regular fashion in the future. A share repurchase tends to increase stock prices because it signals that management thinks the firm’s stock is undervalued. Other effects are an increase in net debt from the use of cash, increasing the tax shield. In addition there is a decrease in the shares outstanding leading to an increase in EPS. Using the previously stated definition of dividend, paying it suggests that the company must, at least, have "excess" cash on hand. Thus, the question boils down to: what should a company do with its "excess" cash? Such a company has two alternatives: either invest the money in projects (internal expansion or acquisitions) or distribute it to shareholders. The choice between these two alternatives is simple: if the company has good projects (i.e., those with NPV > 0), then it should invest in these projects as they would create value to shareholders. Otherwise, the company should distribute the "excess" cash to shareholders in the form of dividends.
Investment Banking Resource Guide Page 90 of 108







Wharton Finance Club, 2007-2008



Thus, when a company increases its regular cash dividend it is typically saying one of two things. One, using the "burden" argument outlined above, the company must be saying (or signaling) that it expects to be profitable in the future and thus the increase in dividend is not a "burden." The second scenario, using the "good projects" argument above, is that the investors may interpret the move as a signal that the company does not have profitable projects and thus it is distributing "excess" cash. Obviously, if the market believes the first argument, then stock prices tend to increase; prices decline if they believe that the company is facing the second scenario. Taxes: Institutional investors do not pay taxes on dividends but individuals do. Individuals can defer capital gains taxes.



10. Describe the difference between issuing a bond at par, discount or a premium. • • Issued at par when the coupon rate equals the market rate. Issued below par (at a discount) when the coupon rate is lower than the market rate. The price of the bonds will be bid down because investors will be receiving an inferior rate. Issued at a premium when the coupon rate is greater than the market rate, which will raise the price of the bond.



11. What is LIBOR and why does it matter? • The London Interbank Offered Rate is the rate that the most creditworthy international banks charge each other for large loans and serves as a basis for other Eurodollar loans to less creditworthy corporate and government borrowers. For instance, a developing country may have to pay one point over LIBOR when it borrows.

12. What major factors affect the yield on a corporate bond? • Corporate bond yields trade at a premium, or spread, over the interest rate on comparable US Treasury bonds. How large this spread is depends on the company’s credit risk: the riskier the company, the higher the interest rate the company must pay to convince investors to lend it money, and therefore, the wider the spread over US Treasuries.

Investment Banking Resource Guide Page 91 of 108

Wharton Finance Club, 2007-2008

13. How would you value a bond, and what interest rate would you use? • A bond value is equal to the present value of its future cash flows (coupon payments and principle repayment) discounted at the prevalent market interest rate. The discount interest rate is dependent on treasury rate plus premium for default/operational risk of issuing entity.

14. What are some different forms of debt? • • • • • • Commercial paper: short-term pure discount obligations issued by the largest and most creditworthy industrial and financial firms Zero coupon bonds Coupon bonds Convertible bonds (hybrid debt) Term loan (basically non-public debt issued by a bank) Many contracts, like leases, are debt-like without actually being debt

15. What type of debt is the most expensive? What type is the least expensive? Aside from cost why might a company choose one type of debt over another? • Senior debt is the cheapest debt for a company to issue and is based on LIBOR+. It guarantees debtholders priority to assets when the firm is in distress. It also tends to have more convenants as it is issued by banks, insurance companies, and other financial institutions. Unsecured debt most expensive to issue as it does not secure any assets to debtholders if the firm is in distress. Unsecured debtholders receive what remains after senior debtholders are paid in full. Unsecured debt tends to have fewer, if any, covenants, is purchased in the public market, and is based on the 10 year Treasury.



16. How do you price a convertible bond? • The owner of a convertible bond owns a bond and a call option on the firm’s stock. The owner must give up the bond to exercise the option. The price of the bond is the higher of the bond value and the conversion value. o o Bond Value at Maturity: Provided the firm maintains a minimum value, then the bond will be worth its face value Conversion at Maturity: If converted, the value increases in proportion to the value of the firm

Investment Banking Resource Guide Page 92 of 108

Wharton Finance Club, 2007-2008

17. What is riskier, equity or debt? • Equity is riskier: it is completely unsecured. In the case of a bankruptcy, all holders of debt have priority over equity holders. That being said, debtholders have no upside potential whereas equity owners do – the reward for being unsecured. How can you do it

18. How can a company reduce its Debt/EBITDA ratio? without increasing EBITDA or paying down debt? • •

Restructure debt to decrease interest expense and hence increase earnings, or reduce debt by either expending cash or issuing equity to pay it down. To do so without paying down debt or increasing EBITDA, you could do with leases and sale-lease back structures.

19. Draw the relationship between WACC and leverage and explain your drawing. • Plotted on the y-axis against leverage on the x-axis, WACC has a U-shape that decreases gradually as leverage increases, passes through an optimal point, and then begins to increase. The decrease is caused by the effects of the interest tax shield as incremental debt (and therefore interest) is added, per the WACC formula:



Past the optimal point, however, WACC increases due to the costs of financial distress that are incurred as the firm’s high leverage increases its probability of default.

Investment Banking Resource Guide Page 93 of 108

Wharton Finance Club, 2007-2008

• •

The smaller the rWACC, the larger the value of the firm. The value of the firm is maximized when rWACC is minimized, hence its optimality. To fully understand these effects, consider Modigliani and Miller Proposition II in a world without taxes or bankruptcy costs:



In this world, rA = rWACC the line is flat, leading to no preference between a firm that is entirely equity financed versus a firm that entirely debt financed.



The reason that rD MM starts to rise above rF is that the debt is now risky in the sense that it has a beta. If the risk of bankruptcy was entirely diversifiable then it would not rise. However, bankruptcy usually occurs in recessions so the debt has market risk and a positive beta. Through the introduction of taxes, the line taxes on an upward vector, indicating the increasing tax shields. In this world, there is no incentive to be anything but debt financed:



Investment Banking Resource Guide Page 94 of 108

Wharton Finance Club, 2007-2008

20. What factors should be considered in determining capital structure? • • • • • WACC minimizing D/E Tax considerations Dilution Perception / Signaling Strategic concerns

21. Why do an IPO over private placement? 22. You're a private company and an investment bank comes to you about taking your company public. What issues do you need to consider? 23. What are three ways an auto manufacturer might raise $500M in capital? Which is the most appropriate? Why? What if it’s a biotech firm that needs to raise the capital rather than an auto manufacturer? 24. Which could take on more debt: a cable company or a business services company? 25. What is a convertible bond? How do you price one? 26. When a bond is issued at a premium what is higher, it’s yield or its coupon? 27. What is the lowest credit rating a company can have before being considered junk?

PRIVATE EQUITY AND LBO
1. What deal metric is most important for PE firms in evaluating deals? • PE firms use IRR to measure performance on an investment. IRR is the break-even discount rate of return when net present value of cash flows is equal to zero. Because the flow of cash over the course of an investment will include occasional calls for additional invested capital as well as occasional dividends, the IRR gives a rate that can be compared across firms. IRR captures both time-weighted and cash flow metrics.

2. What is a leveraged buyout? • A leveraged buyout is the takeover of a company using minimal equity and borrowed funds most often secured by the target company’s assets.
Investment Banking Resource Guide Page 95 of 108

Wharton Finance Club, 2007-2008

• •

The debt tax shield is the “magic” that makes it all work – simply by borrowing the money a higher rate of return is earned. Debt is paid down using the cash flow of the acquired company. Management may use an LBO to retain control by converting a company from public to private. A group of investors may also borrow funds from banks, using their own assets as collateral, to take over another firm. In almost all leveraged buyouts, public shareholders receive a premium over the current market value for their shares. When a company that has gone private in a leveraged buyout offers shares to the public again, it is called reverse leveraged buyout.

3. What multiples are traditionally stated as financial parameters for an LBO? • • • Debt/EBITDA – 4.5-5.5x (up to 6-9x before the recent credit crunch) EBITDA/Interest - > 2x Equity contribution – 25-30% traditionally (although more recently 15-20%)

4. Why lever up a firm? • A major reason for leveraged buyouts is that a target company is for some reason perceived to be undervalued. Thus to gain control of the company, build up the management team, turn around the performance, and sell the company when it has gained a higher value (providing investors with a large premium). LBOs are typically accomplished by either financial buyers (e.g., KKR) or company management, whereas M&A deals are led by companies in the industry (strategic buyers).



5. What are criteria for finding a good LBO candidate? • • • • • • Strong, predictable, steady cash flows o Typically found in mature businesses with low cyclicality o Needed to be able to make interest payments and not default Good management Limited capital expenditures or development needs (low working capital) Undervalued Has a motivated seller Has a viable exit strategy

Investment Banking Resource Guide Page 96 of 108

Wharton Finance Club, 2007-2008

6. If you buy a company for $100 and sell it for $100 two years later, how can you make money? • • Issue yourself dividend payments between purchase and sale Pay down debt during the two years, increasing your equity stake

7. List some various types of exit strategies from an LBO • • • • IPO Spin-offs; break up assets Flip/Sell to strategic buyer MBO

8. What is the transaction process of a typical LBO? • • • • • Deal sourcing Deal screening – strategic due diligence; valuation Secure financing Documentation & filing Bid strategy: break-up fees; MACs (material adverse warranties; mechanics (tender offer vs shareholders vote)

conditions);

9. What is a PIK bond? 10. How do you perform a LBO analysis? building an LBO model. Walk me through the process of

11. Give me an example of Circular Reference in an LBO model. 12. How would you value Wharton as a firm? Is it a possible LBO target? What makes for a good LBO candidate?

ACCOUNTING
1. How do the three main accounting financial statements relate? • The profits that are generated in the income statement get added to retained earnings on the balance sheet. Debt on the balance sheet is used to calculate interest expense on the income statement. Balance sheet has cash balances that are used on the cash flow. Also, cash flow from operations is derived using the changes in balance sheet accounts.

Investment Banking Resource Guide Page 97 of 108

Wharton Finance Club, 2007-2008

2. What line item is usually found on all three financial statements? • Net income

3. What is the difference between a balance sheet, income statement, and statement of cash flows? • A balance sheet describes a firm’s financial status at a specific time (end of fiscal year or quarter). An income statement represents a firm’s operating result during a time period (a fiscal year or quarter). From another angle, a balance sheet tells a business’s economic resources that creditors and shareholders can claim. An income statement summarizes a business’s profitability (revenue minus expenses) within a time period. The statement of cash flows provides more accurate information about certain cash flows than can be inferred from income statements and balance sheets alone – particularly by highlighting the extent to which operations are generating or consuming cash.





4. What is a balance sheet? What are the major line items on it? • The balance sheet is a snapshot that presents the financial position of a company at a given period of time. It lists the economic resources creditors and shareholders can claim. Assets: Resources that a company uses to operate its business o Current Assets: assets that are reasonably expected to be converted to cash within one year in the normal course of business. These are important because they fund day-to-day operations and pay ongoing expenses. Includes: cash, accounts receivable/payable, inventory.





Liabilities: Claims that creditors have on the company’s resources o Current Liabilities: a company’s debts or obligations that are due within one year. Essentially these are bills that are due to creditors and suppliers within a short period of time.



Shareholders’ Equity: The “book value” of a company comes from two main sources: the first source is the original paid in capital, along with any additional investments. The second comes from the Retained Earnings accumulated over time through operations.
Investment Banking Resource Guide Page 98 of 108

Wharton Finance Club, 2007-2008

5. What is an income statement? What are the major line items on it? • • • The income statement provides the results (profitability) of a business’ operations during a specified period of time. Revenues: Source of income that arises from the sale of goods and/or services and is recorded when it is earned. Expenses: Costs incurred by a business over a specified period of time to generate the revenues earned during that same period of time. Commonly includes COGS and SG&A. Net Income: Revenue minus expenses.



6. What are the major line items on the statement of cash flows? • • • • The cash flow provides a summary of the inflows and outflows of cash during a specific time period. Cash Flow from Operations: Includes sales, dividends, interest, cash paid to suppliers, salaries, and taxes. Cash Flow from Investments: Includes purchase and sale of equipment and land. Cash Flow from Financing: Includes repayment of debt and payment of dividends.

7. If you had to use one of the three financial statements to value a company, which would you use and why? • This is a classic interview preparation question – though no one ever seems to actually be asked it. The point of the question is not the right answer, but rather to see your familiarity with the various financial statements and which ones provide which information. A one word answer is woefully inadequate for this question.

8. On which of the three financial statements would you find depreciation? • Cash flow.

Investment Banking Resource Guide Page 99 of 108

Wharton Finance Club, 2007-2008

9. What do you do if you understated depreciation by $100 and discovered the error in a period after the statements are issued? (Assuming 35% tax rate). • Net Income decreases by $65. There is a $100 decrease by taking out the extra $100 in depreciation, but there is a corresponding depreciation tax shield of $35 that partially offsets the decrease. Shareholder’s Equity decreases by $65 due to the decrease in Net Income. Net PPE decreases by $100 due to the increase in depreciation. The Accounting Equation (A = L + SE) is balanced by creating a Deferred Tax Asset of $35. Cash flow remains the same as this is a non-cash transaction. Net Income is reduced by $65, $100 in depreciation is added back, $35 in changes to deferred taxes are subtracted, resulting in no change.





10. If a company decides to use a more accelerated depreciation method (taking $20 in depreciation expense per year instead of $10), walk me through how the three financial statements change? • Same treatment as the example where an extra $100 is found.

11. Describe how the disposal of a fixed asset in exchange for cash would be reflected on a GAAP/IAS statement of cash flows. • In general, disposal of fixed assets is under “net cash flow from investing activities”. The proceeds from the sale are recorded in the Statement of Cash Flows. The gain/loss is recognized on the Income Statement, and the removal of book value of the assets and accumulated depreciation thereof are booked on the Balance Sheet.

12. Walk me through the impact of an asset write-down on the financial statements. • If net book value is greater than maximum of estimate future cash flows or sale value, then the asset is impaired. Calculated impairment loss is equal to net book value minus fair value (market value of asset). Loss is taken on income statement, reducing income, which in turn reduces shareholder equity through reduced retained earnings.

13. In the formula for Net Tangible Assets, are the values for assets and liabilities the market values or book values? • Book values, per GAAP.
Investment Banking Resource Guide Page 100 of 108

Wharton Finance Club, 2007-2008

14. What is goodwill? How is it calculated? How does it affect net income? • Goodwill is the excess of cash paid over the net identifiable assets of the target company as well as the “write-up” of the assets’ historical value. The acquiring company must recognize goodwill as an asset on its financial statements and present it as a separate line item on the balance sheet. Goodwill value = price paid in acquisition – fair market value of tangible assets. Goodwill represents the intangible assets a firm has. For example, the public image / reputation of a firm such as Coca-cola or Nike, and the possible financial value that the image / reputation may add, is considered as goodwill. Goodwill can be generated internally or through acquisition. Goodwill generated internally through the development of a firm is not recognized in the balance sheet. Goodwill can also be generated when an acquisition occurs.

• •



15. What is FAS 142 and what does it say about goodwill? • FAS 142 is a 2001 rule that changed the GAAP treatment of goodwill. As of FAS 142, goodwill is no longer amortized but rather impaired on an annual basis. FAS 142 also ended the “pooling” option for accounting for acquisitions.

16. What are deferred tax assets and liabilities? How do they arise? How do you treat them? • Deferred tax assets (DTAs) arise from paying taxes to the IRS that have not yet been accounted for in accrual terms. Example: if you’ve received a prepayment for services not yet performed and have paid IRS taxes for the cash received, it would create a deferred tax asset because the revenue has not yet been booked on the income statement. Deferred tax liabilities (DTLs) arise from booking taxes in accrual terms prior to payment of tax to IRS. Example: the common practice of using accelerated depreciation (MACRs) for the tax books while using straight-line (or other slower depreciation method) depreciation for the income statement creates DTLs.



Investment Banking Resource Guide Page 101 of 108

Wharton Finance Club, 2007-2008

17. What is working capital? • Working capital is the capital necessary for the company to keep operating. It is defined as current asset minus current liabilities; sometimes called “net working capital” or “net current assets.” The important point to remember in doing problems is that the cash flows associated with working capital are the changes in working capital. When working capital requirements go up, there is a negative cash flow (you need to provide funds); when working capital requirements go down there is a positive cash flow (funds freed up).



18. Using the indirect method, what are the four major adjustments you make to Net Income to arrive at Cash Flow from Operations? • • • • Add depreciation Subtract net increases in Accounts Receivable Subtract net increases in Inventory Add net increases in Accounts Payable

19. If a company changes from LIFO to FIFO, how would that impact its financial statements? • • The effects of inventory costing on financial statements depend on whether costs are increasing or decreasing. Assuming costs are increasing, LIFO to FIFO will increase COGS and thereby decrease Net Income. It will also reduce taxes and inventory. Cash flow increases. If costs are decreasing, the opposite is true.

20. What’s the difference between operating and capital leases? Why would you capitalize an operating lease? How would you do it? • Leases are forms of financing that appear off the balance sheet. For operating leases - those in which the risks and rewards of ownership are not transferred to the lessee - this is acceptable. For capital leases, those in which the risks and rewards of ownership are transferred to the lessee, this requires the lease to be reflected on the balance sheet given that it is essentially a form of debt. A lease is a capital lease if it does any of one of the four following things: (1) transfers ownership at end of lease term (2) contains a bargain purchase option (3) extends for at least 75% of the asset’s life or (4) pays out more than 90% of the fair market value of the asset over its term.
Investment Banking Resource Guide Page 102 of 108



Wharton Finance Club, 2007-2008



Because the obligation associated with an operating lease is not on the balance sheet, an analyst may want to capitalize the lease – in other words, recast the financial statements to reflect the obligations of operating leases and the associated assets. To capitalize an operating lease (1) calculate the present value of the minimum operating lease payments at the current balance sheet date, (2) add the leasehold asset and leasehold obligation to the balance sheet, (3) remove rent expense from the income statement, (4) replace it with amortization and interest expense, (5) adjust cash flows by removing rent expense and adding back interest and amortization. Classify the repayment on the obligation as financing. As a short cut, ratings agencies often multiply operating lease payments by eight in order to come up with an estimate of their value if capitalized.





21. Where would you put a convertible bond on the balance sheet? • Under Long Term Liabilities.

22. Why might a bond’s cash payment and interest expense be different during a given period? • A bond’s cash payment is its coupon rate times its face value. A bond’s interest expense is its market yield times the value of the balance sheet debt liability. The cash payment and interest expense will only be identical if the bond is issued at par because the yield and coupon rates are identical. If the bond is issued at a premium or discount, the rates will be different as will the face value and liability.

23. What are three possible explanations for a declining ROE? • • • Net Income went down. More equity was issued. Dividends were paid out, lowering shareholder’s equity.

24. What is minority interest? • Minority interest is a parent company’s claim to its subsidiary’s earnings, contingent upon what percentage of the subsidiary is owned by the parent company.

Investment Banking Resource Guide Page 103 of 108

Wharton Finance Club, 2007-2008



When the minority interest is significant in absolute amount, parent company’s treatment of minority interests can affect its interest coverage, which is an indicator of a company’s financial healthiness. Although real world treatment varies, the theoretically accurate accounting treatment for the parent company is: o Net earnings should exclude earnings from subsidiaries. o Dividend charges should also exclude those of subsidiaries as well.



25. Say you look at the financial statements of a firm for two consecutive years. Every line item on the income statement has the same value for both years, but the numbers on the lines in the cash flow statement are different for the two years. Speculate on a few things that may have happened to cause this outcome. 26. Imagine you run a hamburger restaurant. Describe the accounting entries for the sale of a hamburger? (Debits and Credits) 27. If I add $10 PPE to a company, walk me through the statements. Now add the depreciation. 28. If $10 of accounts receivable is left out, how would you modify the financial statements? (Assuming everything balanced prior to finding error.) 29. With bonds issued at discounts or premia, what account is debited/credited to balance the difference between the interest expense and the cash payment of the coupon? 30. If a bond is issued with a face value of $100 with a yield of 5% and a coupon of 4%, will the issuer receive greater than, less than, or exactly $100 in cash at the time of sale? 31. What’s the difference between a temporary difference and a permanent difference in taxes? Give an example of each. 32. How do capital leases affect the balance sheet differently from operating leases? The income statement? The statement of cash flows? 33. What is ROE? 34. What is gross margin? Net margin? Operating margin? 35. Name three items included under Other Comprehensive Income. 36. What are three ways retained earnings might go down?
Investment Banking Resource Guide Page 104 of 108

Wharton Finance Club, 2007-2008

37. What happens if inventory turnover increases from 60 days to 75 days? What happens to COGS and Cash Flow statement? 38. What is the problem in comparing LIFO and FIFO companies? 39. Name two possible catalysts for changes in the LIFO reserve. 40. If a company pays back its debt ahead of schedule, what is the impact on the income statement? 41. If convertible debt is converted, what is the impact on the balance sheet? 42. How do you account for stock options during the vesting period? exercise? Upon

MACRO AND MARKETS
1. What is the current Market Risk Premium? • Banks traditionally put this at 4-7%. Currently, banks are using the lower end of the scale.

2. What are the four types of bond yield curves and what do they mean? • Normal Yield Curve: As its name indicates, this is the yield curve shape that forms during normal market conditions, wherein investors generally believe that there will be no significant changes in the economy, such as in inflation rates, and that the economy will continue to grow at a normal rate. During such conditions, investors expect higher yields for fixed income instruments with long-term maturities that occur farther into the future The steep yield curve: Historically, the 20-year Treasury bond yield has averaged approximately two percentage points above that of three-month Treasury bills. In situations when this gap increases (e.g. 20-year Treasury yield rises relatively higher than the three-month Treasury yield), the economy is expected to improve quickly in the future. This type of curve can be seen at the beginning of an economic expansion (right after the end of a recession). Here, economic stagnation will have depressed short-term interest rates; however, rates begin to rise once the demand for capital is re-established by growing economic activity. The inverted yield curve is the least common, but perhaps most important bond yield curve. It means generally signals a recession.
Investment Banking Resource Guide Page 105 of 108





Wharton Finance Club, 2007-2008

3. What relationship do a flat yield curve, long term rates and inflation have? • A flat yield curve indicates that the market environment is sending mixed signals to investors, who are interpreting interest rate movements in various ways. During such an environment, it is difficult for the market to determine whether interest rates will move significantly in either direction farther into the future, therefore the normal “higher inflation” assumptions the further out you are on a yield curve that a normal yield curve brings cannot be made with a flat yield curve. A flat yield curve usually occurs when the market is making a transition that emits different but simultaneous indications of what interest rates will do in the future. When the yield curve is flat, investors can maximize their risk/return tradeoff by choosing fixed-income securities with the least risk, or highest credit quality.

4. What did the Dow close at yesterday? The NASDAQ? The S&P 500? 5. What’s the price of a barrel of oil? 6. What’s the yield on the 10-year bond? The federal funds rate? LIBOR? 7. What was on the front page of the Wall Street Journal this morning? 8. Pretend you are Jeremy Siegel and give me a market briefing. 9. What’s happened in the markets during the past three months? 10. Where do you think interest rates will be in the next 12 months? Where is the bond market going? Where is the equity market going? Where is the foreign exchange market going? 11. Why are lenders willing to provide capital at today's low interest rates? 12. What do you think of Ben Bernanke? Henry Paulson? 13. What is your view on oil prices? Where do you think the price of oil will be in one year? 14. What’s the relationship between unemployment and inflation? 15. If you buy a call option, what are you predicting? 16. Is a strong dollar good or bad for U.S. corporations with operations abroad?
Investment Banking Resource Guide Page 106 of 108

Wharton Finance Club, 2007-2008

BRAINTEASERS
You can’t really prepare for brainteasers. But there are a few common varieties worth examining briefly. At the end are some others that have been asked in the past. 1. The time is 11:08am. Exactly what angle does the large and small hand represent? • • • • • Hour = 30 degrees Minutes = 6 degrees (= 360/60) Hour hand movement as result of minutes = minutes / 2 (= minutes *30/60) Be cognizant of directions in which hands are placed and have moved. Each time the minute hand moves, it moves 6 degrees. 8 minutes is 48 degrees past 12 for the minutes hand. The hour hand has moved a little past 11. Since each minute mark on the clock represents 1/5 of an hour, at 8 minutes it has gone 2/3 of 1/5. In other words, 4 minutes + 1/3 of a minute remain. That’s 4*6 + 6/3 or 26, plus 48 it’s 74.

2. If 50 x 50 can be calculated as (50+10) x (50-10) + 10x10, where 10 is a randomly chosen 'delta', what is 99 x 99? • (98 x 100)+1x1 = 9801

3. If you save $100 in a bank and it doubles in 4 years, what is the interest rate? • • ~ 18% General rule: 72 divided by rate of return equals the number of years required to double an investment’s value. Similarly, 72 divided by number of years needed to double investment value equals the rate of return.

4. If the width of the page is 2mm, what is the width if I fold it 30 times? • 2^30

5. What is the value of a firm with $100M in perpetuity, discount rate of 10% and growth of 5%? 6. What is 11 cubed? 7. What is 43/64 in percentage terms? 8. What is the square root of 0.1? (Obviously, no calculator.)
Investment Banking Resource Guide Page 107 of 108

Wharton Finance Club, 2007-2008

9. If you invested $10, and in 3 years the investment is worth $40, what is the IRR (ignoring taxes)? 10. If I gave you $1 a day for 10 years or $1000 today which one will you choose? 11. Now if I gave you $1 everyday for life and $1000, which one would you choose? 12. If I offered you $1M today and I took 10% of your annual lifetime earnings, would you take it? 13. You have $10K in credit card debt with 10% interest rate. You have $10K in stocks earning 10% per year. Do you liquidate your stocks to pay down the debt? 14. Given two hourglass timers full of sand (one for 9 minutes, one for 7 minutes) how do you time 11 minutes? 15. Why London's tube escalators have two up and one down (more escalators operating upward than downward)? 16. Why are manhole covers round?

Investment Banking Resource Guide Page 108 of 108

Sponsor Documents

Or use your account on DocShare.tips

Hide

Forgot your password?

Or register your new account on DocShare.tips

Hide

Lost your password? Please enter your email address. You will receive a link to create a new password.

Back to log-in

Close